GENERAL PRACTITIONER EXAM
Question Summary
0 of 100 questions completed
Questions:
- 1
- 2
- 3
- 4
- 5
- 6
- 7
- 8
- 9
- 10
- 11
- 12
- 13
- 14
- 15
- 16
- 17
- 18
- 19
- 20
- 21
- 22
- 23
- 24
- 25
- 26
- 27
- 28
- 29
- 30
- 31
- 32
- 33
- 34
- 35
- 36
- 37
- 38
- 39
- 40
- 41
- 42
- 43
- 44
- 45
- 46
- 47
- 48
- 49
- 50
- 51
- 52
- 53
- 54
- 55
- 56
- 57
- 58
- 59
- 60
- 61
- 62
- 63
- 64
- 65
- 66
- 67
- 68
- 69
- 70
- 71
- 72
- 73
- 74
- 75
- 76
- 77
- 78
- 79
- 80
- 81
- 82
- 83
- 84
- 85
- 86
- 87
- 88
- 89
- 90
- 91
- 92
- 93
- 94
- 95
- 96
- 97
- 98
- 99
- 100
Information
Hi, Welcome to General Practitioner Exam
You have already completed the Exam before. Hence you can not start it again.
Exam is loading...
You must sign in or sign up to start the Exam.
You have to finish following Exam, to start this Exam:
Results
0 of 100 questions answered correctly
Your time:
Time has elapsed
You have reached 0 of 0 points, (0)
Average score |
|
Your score |
|
Categories
- Not categorized 0%
Pos. | Name | Entered on | Points | Result |
---|---|---|---|---|
Table is loading | ||||
No data available | ||||
- 1
- 2
- 3
- 4
- 5
- 6
- 7
- 8
- 9
- 10
- 11
- 12
- 13
- 14
- 15
- 16
- 17
- 18
- 19
- 20
- 21
- 22
- 23
- 24
- 25
- 26
- 27
- 28
- 29
- 30
- 31
- 32
- 33
- 34
- 35
- 36
- 37
- 38
- 39
- 40
- 41
- 42
- 43
- 44
- 45
- 46
- 47
- 48
- 49
- 50
- 51
- 52
- 53
- 54
- 55
- 56
- 57
- 58
- 59
- 60
- 61
- 62
- 63
- 64
- 65
- 66
- 67
- 68
- 69
- 70
- 71
- 72
- 73
- 74
- 75
- 76
- 77
- 78
- 79
- 80
- 81
- 82
- 83
- 84
- 85
- 86
- 87
- 88
- 89
- 90
- 91
- 92
- 93
- 94
- 95
- 96
- 97
- 98
- 99
- 100
- Unanswered
- Answered
- Review
-
Question 1 of 100
1. Question
1 pointsA 35 year old man suffered from a car accident. He is admitted to the hospital with a fracture of the tenth rib on the left side. He is pale, restless, and sweating, and his blood pressure is low. There is pronounced guarding in the left upper quadrant of the abdomen on examination. Which of the following viscera may be damaged?
Correct
Incorrect
Explanation:
Significant impact (eg, motor vehicle collision) can rupture a normal spleen. Rupture of the splenic capsule produces marked hemorrhage into the peritoneal cavity. The manifestations, including hemorrhagic shock, abdominal pain, and distention, are usually clinically obvious. However, splenic trauma can also produce a subcapsular hematoma, which may not rupture until hours or even months after the injury.
Rupture is generally preceded by left upper quadrant abdominal pain. Splenic rupture should be suspected in patients with blunt abdominal trauma and hemorrhagic shock or left upper quadrant pain (which sometimes radiates to the shoulder); patients with unexplained left upper quadrant pain, particularly if there is evidence of hypovolemia or shock, should be asked about recent trauma. The diagnosis is confirmed by CT scan (in the stable patient), ultrasound, or peritoneal lavage (in the unstable patient).
Treatment has traditionally been splenectomy. However, splenectomy should be avoided if possible, particularly in children, to avoid the resulting permanent susceptibility to bacterial infections. -
Question 2 of 100
2. Question
1 pointsA 70 year old Asian male presents with hematochezia. He has stable vital signs. Lower endoscopy is performed, but is unsuccessful due to active bleeding. Which appropriate investigation would follow next?
Correct
Incorrect
Explanation:
In most patients with heavy gastrointestinal bleeding, localizing the bleeding site, rather than diagnosing the cause of the bleeding, is the most important task. A lower GI series is usually nondiagnostic during heavy, active bleeding. A small-bowel radiograph may be helpful after the active bleeding has stopped, but not during the acute phase of the bleeding. A blood pool scan allows repeated scanning over a prolonged period of time, with the goal of permitting enough accumulation of the isotope to direct the arteriographer to the most likely source of the bleeding. If the scan is negative, arteriography would also be unlikely to reveal the active source of bleeding. It is also a more invasive procedure. Exploratory laparotomy may be indicated if a blood pool scan or an arteriogram is nondiagnostic and the patient continues to bleed heavily. -
Question 3 of 100
3. Question
1 pointsA female aged 70 is found to have a complex splenic mass on a CT scan performed for another reason. Because of concerns regarding possible malignancy and the potential for spontaneous splenic rupture, an elective splenectomy is planned. Which one of the following immunizations should be given prior to her surgery?
Correct
Incorrect
Explanation:
Asplenic and functionally asplenic patients are susceptible to overwhelming infection from encapsulated organisms, and should be immunized with pneumococcal, meningococcal, and Hib vaccines. The vaccines´ immunogenicity may be reduced when given after splenectomy or during chemotherapy; thus, they should be given at least 2 weeks before performing elective splenectomy or starting chemotherapy if possible.
-
Question 4 of 100
4. Question
1 pointsA man aged 25 presents with a history of recent bilious vomiting followed by a large volume of bright red hematemesis. He is pale, sweaty and anxious and complains of mild epigastric pain. Blood pressure is 80/50 mm Hg, pulse 128/minute. All of the following steps are correct, except
Correct
Incorrect
Explanation:
Esophaegal varices are dilated veins in the distal esophagus or proximal stomach caused by elevated pressure in the portal venous system, typically from cirrhosis. They may bleed massively but cause no other symptoms. Diagnosis is by upper endoscopy.
Treatment involves fluid resuscitation, endoscopic banding (sclerotherapy second choice) IV octreotide and possibly a transjugular intrahepatic portosystemic shunting (TIPS) procedure.
If bleeding continues or recurs despite these measures, emergency techniques to shunt blood from the portal system to the vena cava can lower portal pressure and diminish bleeding. A TIPS procedure is the emergency intervention of choice. TIPS is an invasive radiologic procedure in which a guidewire is passed from the vena cava through the liver parenchyma into the portal circulation.
The resultant passage is dilated by a balloon catheter, and a metallic stent is inserted, creating a bypass between the portal and hepatic venous circulations. Stent size is crucial. If the stent is too large, hepatic encephalopathy results because of diversion of too much portal blood flow from the liver. If the stent is too small, it is more likely to occlude. -
Question 5 of 100
5. Question
1 pointsA white mail patient aged 58 years has been in ICU for the past 10 days recovering from an exploratory laparotomy performed for a perforated duodenal ulcer. The patient developed acute renal failure and sepsis postoperatively. 3 days ago the patient became hypothermic and the resident on duty ordered a thyroid function tests and obtained the following results:
T4RIA …………………………….. 4 microg/dL (N 5-12)
T3RIA ………………………………. 60 n/dL /N 70-190)
TSH ………………………………… 2.0 microU/mL (N 0.5-5.0)No past history of thyroid disease is present. His gland is normal in size. His condition today is critical but stable. The most appropriate management at this time is to
Correct
Incorrect
Explanation:
This patient in all probability has the euthyroid-sick syndrome, also known as nonthyroid illness syndrome (NTI)-the association of severe nonthyroidal illness with biochemical parameters indicative of thyroid hypofunction. Low T3 with normal T4 and low T3 with low T4 are the most common variants of this syndrome. TSH is usually normal but may be high or low. A TSH level > 20 microU/mL would be inconsistent with NTI and ´indicates hypothyroidism. Replacement of T4 does not influence the outcome.
-
Question 6 of 100
6. Question
1 pointsA man presents with an abscess situated in the right posterior triangle. After its surgical incision the patient could no longer raise her right hand above her head to brush her hair. Which one of the following nerves is injured in this patient?
Correct
Incorrect
Explanation:
The accessory nerve (CN XI) runs through the posterior triangle, so it is likely to be injured in posterior neck triangle operations. If the accessory nerve is damaged there, the trapezius will be denervated, the tip of the shoulder will be drooping and the patient will have inability to raise her hand above her head.
-
Question 7 of 100
7. Question
1 pointsA lady aged 39 year presents to you with a 4 cm palpable nodule in her right breast. Fine-needle aspiration yields 4 cc of bloody fluid. The breast nodule is no longer palpable following aspiration. Which strategy would be most appropriate at this point?
Correct
Incorrect
Explanation:
When straw-colored or gray-green fluid is obtained by fine-needle aspiration of a breast nodule and the lesion completely disappears, the diagnosis is simple cyst. The fluid should not be sent for analysis because the risk for cancer is exceedingly small. If the fluid is bloody or otherwise unusual, it should be sent for cytologic examination because about 7% of bloodstained aspirates are associated with cancer.
-
Question 8 of 100
8. Question
1 pointsA 40 year-old has fever and diarrhea with abdominal pain and tenderness. She has a highly dilated abdomen and is jaundiced along symptoms consistent with encephalopathy. Abdominal X-rays reveal free air under the diaphragm.
The laboratory values are as follows:Polymorphonuclear cell count: 300/ ml (ascitic fluid) Gram stain: Gram-negative rods Lacate: 36 mg/dl Glucose (blood): 69 mg/dl Lactate dehydrogenase: 200mU/mL Which one of the following represents the most likely diagnosis?
Correct
Incorrect
Explanation:
The presence of free air under the diaphragm should be almost pathognomic for the diagnosis of secondary bacterial peritonitis, which most often results from the perforation of the bowel wall. Non-ruptured duodenal and gastric ulcers will not reveal free air under the diaphragm on a vertical chest x ray. In addition, the abdomen will not be dilated. All of the laboratory findings are consistent with the diagnosis of secondary peritonitis. Esophageal rupture is rarer as compared to bowel perforation and more likely to produce air in the chest cavity.
-
Question 9 of 100
9. Question
1 pointsA 58-year-old woman presents with symptoms of weight loss, depression, and painless jaundice. On examination, she has hepatomegaly and ascites. Which is the most sensitive test to diagnose pancreatic head carcinoma?
Correct
Incorrect
Explanation:
ERP is an invasive procedure, but is the most sensitive diagnostic test to localize carcinoma at the head of the pancreas. Carcinoembryonic antigen is a serum-tumor marker which is used also in fo1low-up after colon cancer surgery. It is insensitive and non-specific. CA 19-9 is also a serum-tumor marker and not sensitive enough to diagnose pancreatic cancer. Ultrasound of the abdomen is a useful screening test. It detects abnormalities 75% of the time. It can detect tumors up to 2 cm in size. Upper endoscopy can detect tumors and polyps in the stomach, duodenum, and jejunum. -
Question 10 of 100
10. Question
1 pointsIn advising a patient diagnosed with breast carcinoma with positive nodes, which one of the following statements is correct?
Correct
Incorrect
Explanation:
While recent technological advances have led to earlier detection of breast cancer, this has not led to an absolute cure and a marked increase in survival. The major advance has been in demonstrating that lumpectomy with axillary node dissection offers about the same five-year survival as mastectomy The choice between these treatments should be determined on an individual treatment basis, with the psychological implications of mastectomy weighed against the fear of recurrence if the breast is not removed
-
Question 11 of 100
11. Question
1 pointsPhysical examination of a 36 year old man´s shaft of penis shows a solitary, thickened, whitish plaque with a slightly ulcerated, crusted surface. Biopsy reveals markedly dysplastic epithelial cells, many mitoses (some of which are abnormal), disordered epithelial maturation, and an intact basement membrane with no evidence of stromal invasion. The most likely diagnosis is
Correct
Incorrect
Explanation:
This is Bowen´s disease, which is one clinical form of penile carcinoma in situ (the other forms are Bowenoid papulosis and Erythroplasia of Queyrat). Untreated Bowen´s disease will, over a period of years, progress to frank invasive carcinoma in 10% to 20% of patients. Bowen´s disease can affect the genital region of both men and women. Bowenoid papulosis is another form of penile carcinoma in situ, characterized clinically by multiple reddish-brown papillary lesions. Condyloma acuminatum usually produces a papillary lesion. Erythroplasia of Queyrat is another form of penile carcinoma in situ, characterized by single or multiple shiny red plaques. Squamous cell carcinoma will be frankly invasive on biopsy. -
Question 12 of 100
12. Question
1 pointsA 53 year old man presents with jaundice. Ultrasonography demonstrates a 5 cm mass in the head of the pancreas. Endoscopic retrograde cholangiopancreatography with cytologic sampling demonstrates cells with large hyperchromatic nuclei and a high nuclear/ cytoplasmic ratio. Few small glands composed of these cells are also seen in the cytologic preparation. The prognosis of this condition would be most similar to which condition?
Correct
Incorrect
Explanation:
The patient probably has pancreatic adenocarcinoma. This cancer carries one of the worst prognoses with a 3.5% overall 5-year survival rate despite all attempts at aggressive management. The prognosis is also bleak with adenocarcinoma of the esophagus, with a 10% overall 5 year survival rate. Adenocarcinoma of the breast now has an overall 5 year survival rate of 60-70%. Adenocarcinoma of the colon now has an overall 5-year survival rate of 50-60%. Adenocarcinoma of the prostate now has an overall 5-year survival rate of 50-70%. Primary gastric lymphoma has an overall 5 year survival rate of 75-85 %.The patient probably has pancreatic adenocarcinoma. This cancer carries one of the worst prognoses with a 3.5% overall 5-year survival rate despite all attempts at aggressive management. The prognosis is also bleak with adenocarcinoma of the esophagus, with a 10% overall 5 year survival rate. Adenocarcinoma of the breast now has an overall 5 year survival rate of 60-70%. Adenocarcinoma of the colon now has an overall 5-year survival rate of 50-60%. Adenocarcinoma of the prostate now has an overall 5-year survival rate of 50-70%. Primary gastric lymphoma has an overall 5 year survival rate of 75-85 %. -
Question 13 of 100
13. Question
1 pointsA 56 year old man complains of impotence of duration of two years. What is the most likely organic cause?
Correct
Incorrect
Explanation:
Organic cases of impotence are uncommon. In this patient without history or evidence of diabetes or drug exposure, a pituitary tumor is the most common organic cause. Primary hypogonadism is rare and is usually accompanied by other complaints (e.g., weakness). Neuropathies, not secondary to diabetes mellitus, are rare and patients usually present with complaints of pain, numbness, or weakness.
-
Question 14 of 100
14. Question
1 pointsFive patients visit the hospital with Five different cancers. Which will respond best to adjuvant chemotherapy?
Correct
Incorrect
Explanation:
Breast cancer responds the best of all these tumors to the use of adjuvant chemotherapy after surgical resection.
-
Question 15 of 100
15. Question
1 pointsA 33 year old man is in the ICU following a motor vehicle accident. He is in a coma and is receiving enteral feedings through a jejunostomy tube. Diarrhea starts to occur after several days. All of the following are associated with diarrhea secondary to enteral feedings, EXCEPT
Correct
Incorrect
Explanation:
Rapid bolus feedings in the supine position in a comatose patient can cause aspiration of the formula, and not diarrhea. Hypertonic feedings cause high osmolality and thus osmolar diarrhea. Bacterial contamination of feedings can cause infectious diarrhea. Refrigeration of formula and strict handwashing is necessary. Decrease in the acidity of the bowel by H2 blockers can alter the bowel flora and thus cause diarrhea. Antibiotics can also alter the bowel flora and cause C. difficile diarrhea.
-
Question 16 of 100
16. Question
1 pointsDuring surgery of a 47 year old woman, the surgeon finds cystic masses in both ovaries. Each mass consists of a unilocular cyst containing clear fluid. The entire wall is covered with papillary excrescences. The most likely diagnosis is
Correct
Incorrect
Explanation:
Similar to testicular tumors, ovarian tumors can be classified according to cell of origin. There are three main categories: tumors of surface epithelium, tumors of germ cell origin, and tumors of sex cord stromal origin. Ovarian surface epithelium (coelomic mesothelium) may differentiate along tubal (serous), cervical (mucinous), or endometrial lines giving origin to serous cystadenoma/cystadenocarcinoma, mucinous cystadenoma/cystadenocarcinoma and endometrioid tumors, respectively. Serous tumors represent 40% of all ovarian tumors, and serous cystadenocarcinoma is the most frequent serous tumor. Serous cystadenocarcinomas occur primarily in women aged 40 to 65 years. About two thirds of these tumors are bilateral. As the name suggests it is a cystic tumor containing clear fluid. The cystic wall is lined by malignant epithelial cells forming papillary fronds. Less frequent than serous cystadenocarcinoma, endometrioid adenocarcinoma and mucinous cystadenocarcinoma also derive from surface epithelium. Endometrioid carcinoma is histologically similar to endometrial adenocarcinoma, whereas mucinous cystadenocarcinoma is composed of mucin producing cells similar to cervical epithelium. Both these neoplasms have solid and cystic areas (mucinous cystadenocarcinoma more so than endometrioid carcinoma) and may be bilateral (endometrioid carcinoma more frequently than mucinous cystadenocarcinoma). Granulosa cell tumors originate from ovarian stroma and consist of variable mixtures of granulosa cells and theca cells. Since they frequently produce large amounts of estrogens, these tumors manifest with precocious puberty in preadolescent girls. On the contrary, mature women with granulosa cell tumors develop endometrial hyperplasia and fibrocystic change of breast. Histologically, these neoplasms are composed of uniform cuboidal cells, forming structures reminiscent of ovarian follicles (Call-Exner bodies). Mature cystic teratoma is the most frequent neoplasm derived from germ cells. Teratomas can be further classified into mature cystic, immature, and monodermal teratomas. The great majority of teratomas are mature cystic. Since they originate from more than one germ layer, these neoplasms contain an amazing mixture of mature tissue components, often including skin, teeth, neural epithelium, thyroid, cartilage, and intestinal tissue, for example.
-
Question 17 of 100
17. Question
1 pointsA 28 year old man undergoes surgery for resection of a segment of terminal ileum. He had low grade fever, weight loss, and symptoms of intestinal occlusion. The surgical specimen is shown below. The most likely diagnosis is
Correct
Incorrect
Explanation:
The combination of the clinical history and macroscopic features of the surgical specimen is consistent with Crohn disease. Histologic examination of this segment would likely reveal transmural chronic inflammation and scattered noncaseating granulomas. This chronic inflammatory condition may affect the whole alimentary tract, from mouth to anus, but most commonly involves the terminal ileum. The most important clues to the correct choice (clues that also guide the clinical diagnosis) include sharp demarcation of involved segments of bowel and the involvement of the terminal ileum. The young age of the patient is also consistent with the epidemiologic features of Crohn disease. Adenocarcinoma of the colon develops most commonly alter 60 years of age. The small intestine is not usually involved. The gross picture would show either a fungating, often ulcerated mass protruding into the lumen; or a ring like infiltrating lesion that narrows the intestinal lumen. Diverticular disease is particularly prevalent in elderly people, and usually involves the distal colon. Involvement of the terminal ileum is unlikely. Asymptomatic cases are more frequent than those giving rise to clinical manifestations. Grossly, you would see outpouchings of the mucosa covered by a thin layer of serosa without a muscular layer. Intestinal tuberculosis is now a rare condition, occasionally encountered in immunosuppressed patients. Ulcerations of the mucosa are the predominant macroscopic features. Caseating granulomas are the most typical histologic features. Ulcerative colitis overlaps with Crohn disease in clinical and histopathologic features. This overlap justifies the “umbrella” designation of inflammatory bowel disease, used by clinicians and pathologists when differentiation between these conditions is not possible (10-15% of cases). However the presence of “skip” segments between involved intestinal loops and non necrotizing granulomas on histologic examination favors a diagnosis of Crohn disease over that of ulcerative colitis. The terminal ileum is only involved in ulcerative colitis if there is extensive disease through the whole length of the large bowel.
-
Question 18 of 100
18. Question
1 pointsA 63 year old man has had midabdominal colicky pain, with nausea, anorexia, and vomiting for the past 2 days. He has a history of DVT but he is not currently on anticoagulants. His temperature is 98.6oF, BP is 95/60 mm Hg, and pulse is 102/min. He has a distended and tympanitic abdomen. Abdominal X-ray shows multiple distended loops of small bowel and distension of the right colon, up to the middle of the transverse colon. Stool is positive for occult blood. The next step in the evaluation is
Correct
Incorrect
Explanation:
Mesenteric ischemia is caused by a reduction in intestinal blood flow, which most commonly arises from occlusion, vasospasm, and/or hypoperfusion of the mesenteric vasculature. A CT scan of the abdomen (or MRI, which was not offered as a choice) has excellent diagnostic yield when looking for mesenteric venous thrombosis. The clinical picture is indeed very suggestive for that condition. Colonoscopy (A) or sigmoidoscopy is often required to establish the diagnosis of ischemic colitis. They are usually performed without bowel preparation to avoid reducing blood flow from dehydrating cathartics), and with minimization of air insufflation in an effort to avoid overdistention and perforation. Colonoscopy is usually reserved for occasions where CT is inconclusive. Color Doppler ultrasound (C) also has excellent diagnostic capability if performed by an experienced sonographer, under ideal circumstances. Gas, however, is the “enemy´ of sonograms. Given the gas distention in this man, the circumstances are no longer ideal for the ultrasound. Exploratory laparotomy (D) is in order if there are signs of peritonitis (which this mar does not have). Surgical treatment is not necessarily imperative at this time. The peritonitis could still be managed with anticoagulants or thrombolytic agents. Laparoscopy (E) is less invasive than open laparotomy, but the necessary insufflation of the abdomen in an already distended individual, with marginal vital signs, is not a good idea particularly when the noninvasive CT scan can enable the physician to make the diagnosis.
-
Question 19 of 100
19. Question
1 pointsAn alcoholic and diabetic aged 56 years has several years of intense epigastric pain, present at all times, and exacerbated by eating. He also has oily spots on stool surface. He began to have acute episodes of severe upper abdominal pain in his mid thirties. He has been smoking 1 pack a day for past 30 years. His BMI is 18 and he has upper abdominal tenderness. The accurate test for diagnosis is
Correct
Incorrect
Explanation:
The clinical diagnosis is chronic pancreatitis because of the history of intense epigastric pain and long-standing heavy alcohol intake. Steatorrhea and diabetes reflect the pancreatic insufficiency. ERCP (C) provides the most accurate visualization of the pancreatic ductal system and is regarded as the diagnosing criterion of chronic pancreatitis. ERCP is not the best initial test as it is invasive, expensive, and carries a risk (operator-dependent) of pancreatitis. CT scan (B) is the best initial step as it gives useful information about potentially correctable features of the disease, such as pseudocysts, dilated ducts, calculi, or areas amenable to resection. It is not the most accurate test because the subtle abnormalities of early to moderate chronic pancreatitis are beyond its resolution, and a normal finding on this study does not rule out chronic pancreatitis. This study is indicated to look for complications of the disease. Arteriogram (A) adds little to the diagnosis of chronic pancreatitis, and it is invasive and expensive. Sonogram (D) lacks the necessary detail for this evaluation. It is, however, an excellent way to follow the size of pseudocysts that are being observed. Upper gastrointestinal series (E) would give minimal information about the pancreas.
-
Question 20 of 100
20. Question
1 pointsA 54 year old male is brought with multiple abdominal blunt traumas after an accident. After successfully repairing all intra-abdominal injuries, the abdomen was closed. On 10th post-op day he experienced temperature spikes. Exam shows fresh midline wound and some mild tenderness in the periumbilical area. WBC count is 18,400/mm3. Which study will show the abnormality responsible for the fever?
Correct
Incorrect
Explanation:
Fever that starts about 10 to 15 days after a contaminated abdominal surgical procedure is most likely due to a deep abscess, either pelvic or subphrenic, which is further supported by the elevated WBC. The former has been ruled out by the rectal exam performed as part of his postoperative physical examination; the latter can best be demonstrated by CT scan. A chest X-ray (A) could be helpful if an air-fluid level was to show up beneath the right diaphragm, but such finding is rare. An elevated hemidiaphragm with pleural effusion would also be suggestive, but not diagnostic. The lung itself should not be the source of this late post operative fever. Plain films of the abdomen (C) might show the same findings detailed above, but the chances are low. Pus alone will not be seen in a plain film.
Blood cultures (D) might identity a type of flora, which would suggest the source of the infection (in this case the gastrointestinal tract flora), but would not pinpoint the location of the abscess. X-ray studies of the bowel (E) might identify a leak in an anastomosis and might be a good idea if a gastric or a colonic suture line were suspected, which is not applicable here. -
Question 21 of 100
21. Question
1 pointsA 46 year old female has bone pain, myalgias, constipation, and a depressed mood. She has been hospitalized twice in the last year for urolithiasis. Labs reveal hypercalcemia and elevated PTH level. Her renal function is normal, and neck ultrasound reveals a parathyroid mass. Which laboratory abnormality would be expected in this patient?
Correct
Incorrect
Explanation:
The patient described is suffering from primary hyperthyroidism. The most common cause of primary hyperthyroidism is a parathyroid adenoma; parathyroid carcinoma and hyperplasia are much less common. This condition most commonly occurs in women over the age of 40 years as a solitary adenoma. Genetic syndromes such as MEN I, IIa, and IIb may also be associated with parathyroid adenomas, among other findings. Typical presenting complaints are those associated with hypercalcemia. These include recurrent urolithiasis, constipation, myalgias, bone pain, polyuria, fatigue, and psychologic changes. Patients may be asymptomatic at presentation, however, with the diagnosis made following discovery of an elevated calcium level on routine blood tests. Laboratory studies typically reveal an inappropriately elevated parathyroid hormone (PTH) level, hypercalcemia, hypophosphatemia, and elevated urine calcium. Chronically elevated PTH levels can cause defective bone mineralization and remodeling, leading to osteitis fibrosa cystica, a condition characterized by the formation of “brown tumors” in bone, particularly in the jaw. In cases in which the bone is significantly affected, the serum alkaline phosphatase is elevated, not decreased (E). Hypernatremia (A) typically occurs in the setting of excessive free water loss and is not associated with primary hyperparathyroidism. Hyperphosphatemia (B) and low levels of dihydroxy (24, 25(OH) 2) vitamin D occur in the setting of renal failure/renal osteodystrophy, one cause of secondary hyperparathyroidism. Hyponatremia (C) can occur in the syndrome of inappropriate antidiuretic hormone (SIADH), which is a paraneoplastic condition associated with small cell carcinoma of the lung. Another paraneoplastic syndrome associated with this malignancy is hypercalcemia caused by production of PTH related peptide by the malignant cells, resulting in hypercalcemia and low PTH levels. -
Question 22 of 100
22. Question
1 pointsA 58 year old postmenopausal female noticed a lump in her left breast during self examination 2 days ago. She has no weight loss, bone pain, or anorexia. On exam her left breast has a palpable nodule in the upper outer quadrant. Mammography shows a speculated 8 mm density in same quadrant. Ultrasound guided biopsy shows an invasive ductal carcinoma that is estrogen receptor-positive, with a histology grade of 2 (out of 3). Which treatment regimen is appropriate at this time?
Correct
Incorrect
Explanation:
This patient has stage l breast cancer clinically, based on the size of the lesion and the lack of nodal involvement. Lumpectomy (breast conservation) has been shown to be as efficacious as radical mastectomy when combined with radiation therapy. Lumpectomy alone (A) is associated with higher rates of recurrence in patients with breast cancer. Lumpectomy alone can be performed in some women who fear cosmetic deformity but it is not recommended, due to the high rate of recurrence. Tamoxifen ought to be used if the tumor is estrogen receptor-positive, as it is in this case. Tamoxifen works as a competitive inhibitor of estrogen. It has been shown to reduce the recurrence and mortality when combined with radiation and lumpectomy (C) in this population of patients. Anastrozole (B) is also approved for the treatment of breast cancer in postmenopausal women; however, it has been shown to increase the risk of fractures. Since this patient has a history of osteoporosis, anastrozole is not preferred. Raloxifene, another selective estrogen receptor modulator (SERM) approved for the treatment of breast cancer, has been shown to decrease the risk of fractures in postmenopausal women and therefore can also serve as a preventive agent.
-
Question 23 of 100
23. Question
1 pointsA 27 year old woman is brought by her boyfriend with vague, right upper quadrant abdominal pain. Then she began feeling weak and vomited. Her BP is 81/42 mm Hg and pulse is 121/min. Abdomen is mildly distended and diffusely tender. Hb is 8.1 g/dL. AXR shows no evidence of pneumoperitoneum. She and her boyfriend deny any chance of pregnancy. She is compliant with her birth control pills and her last menses was normal. The source of her condition is
Correct
Incorrect
Explanation:
Ninety percent of hepatic adenomas arise in women aged 20 to 40 years, most of whom use oral contraceptives. The risk of hepatic adenoma seems to be related to the dosage and duration of use of estrogens. It is also associated with glycogen storage disease type 1 and the use of anabolic steroids. It is important to know that these adenomas have a tendency to rupture and bleed massively inside the abdomen. CT scan is diagnostic and emergency surgery is required.
Acute cholecystitis (A) occurs with inflammation and gallbladder distention. Patients will complain of pain consistent with biliary, colic and may have a fever and mild tachycardia. They do not have anemia. Ascending cholangitis (B) is a potentially deadly disease in which gallstones have reached the common duct, producing partial obstruction and ascending infection. Patients are usually much older and very sick. Ectopic pregnancy (C) presents as lower abdominal pain and irregular or absent menstrual periods. This patient has neither. Perforated duodenal ulcer (D) is usually accompanied by sudden and severe epigastric pain with radiation to the right scapula. Physical examination reveals a low grade fever, diminished bowel sounds, and rigid abdominal muscles. Upright abdominal X-ray may reveal pneumoperitoneum, which is not present in this patient. -
Question 24 of 100
24. Question
1 pointsA 63 year old chronic alcoholic and diabetic farmer has chronic right sided otitis media and difficulty swallowing. Multiple courses of antibiotics have failed to resolve his ear infection. Exam reveals a posterior cervical mass on lymph node palpation, and oral mucosa exam reveals an ulcerated mass in the right posterior nasopharynx, biopsy of which reveals squamous cell carcinoma. Which one likely contributed to the development of his disease?
Correct
Incorrect
Explanation:
The patient described has a squamous cell carcinoma (SCC) of the pharynx. SCC of the head and neck has been associated with alcoholism, chronic tobacco use, and infection with the human papilloma virus (HPV), particularly monogenic HPV strains such as type 16. Oropharyngeal SCC most commonly affects men over the age of 50 years, though men may be more likely to abuse alcohol and tobacco, and the most commonly involved sites are the palate and tongue. Lesions classically begin as red or white plaques (erythroplakia and leukoplakia, respectively), but tumors in the nasopharynx and or pharynx often do not become clinically evident until disease is severe (stage III to IV). Lesions of the nasopharynx may occlude the Eustachian tube, causing accumulation of mucus in the middle ear, which is then predisposed to repeated, chronic infections. Aflatoxin (A) is produced by some species of Aspergillus and typically occurs in locations where crop grains are stored. It is associated with the development of hepatocellular carcinoma. Arsenic (C) exposure is associated with squamous cell carcinoma of the skin, but it does not cause oropharyngeal SCC. Obesity (E) is associated with a greater risk for diabetes, stroke, and coronary artery disease and has a detrimental effect on expected lifespan, but it is not directly associated with any particular malignancies.
-
Question 25 of 100
25. Question
1 pointsA patient in a state of shock with multiple traumatic injuries is taken to the operating room for an exploratory laparotomy which lasts for approximately 4 hours. Several liters of Ringer lactate have been infused during surgery. On 2nd post-op day he develops confusion, lower extremity edema and cyanosis, and abdomen distension, and the abdominal sutures are cutting through the skin around them. His creatinine has doubled. At the time of surgery, which treatment option could have prevented these complications?
Correct
Incorrect
Explanation:
This patient has abdominal compartment syndrome .The massive infusion of Ringer lactate, though life-saving, produces severe edema in the surgical field. Forced closure would increase the intra abdominal pressure, compressing the lungs, thus producing hypoxia, and the inferior vena cava, causing renal failure. Distension of the skin will cause the sutures to cut through the skin around them. To avoid this set of complications, a temporary plastic coverage or an absorbable mesh that can be left in place, should be placed at the time of the surgery to allow the bowel to be protected without the undue pressure. Closing the skin only (A) can be life-saving when hypothermia develops during surgery. In this setting, however, the skin will not come to gather without undue tension.
Forced closure (B) would compromise ventilation and produce acute renal failure due to pressure on the inferior vena cava. Diuretics (C) cannot selectively remove the fluid from the swollen tissues. Leaving the bowel exposed to the air (D) is not an option. In the short term, the patient would suffer severe heat loss; later, the bowel would dry out and perorate. -
Question 26 of 100
26. Question
1 pointsA 67 year old woman presents to the ER with an incarcerated umbilical hernia. She has gross abdominal distention, is dehydrated, and reports persistent fecaloid vomiting for the past 3 days. She is tired, weak, and thirsty, but is awake and alert. Labs reveal a sodium concentration of 118mEq/L. She does not have any chronic medical conditions. The physiologic explanation for the serum sodium is
Correct
Incorrect
Explanation:
Gastrointestinal tract fluids have a sodium concentration very close to that of plasma; as they are lost (internally or externally), they should be replaced with isotonic, sodium containing fluids. But that is not what patients typically do at home. Thirsty and unable to eat solid (sodium-containing) foods, they drink water, Coke, and tea, fluids without significant amounts of sodium, which the body avidly retains because of the severe volume depletion. Endogenous water from catabolic activity is also retained. Dilutional hyponatremia eventually develops. She does not have “water intoxication” (A). This term denotes abnormal water retention due to excessive water infusion at a time when there is a high level of ADH in the blood. This patient is retaining water because she is desperately volume depleted, not because high volumes of water are being forced into her.
The hyponatremia is not due to the loss of hypertonic fluid (B). There are no hypertonic fluids in the gut or anywhere else for that matter. The only hypertonic fluid that we can lose is highly concentrated urine, but we usually do so as a physiologic response to save water. Yes, we often see comatose and convulsing patients when they have this much hyponatremia (D), but that happens when water retention is massive and fast. Slow water retention allows the brain to adapt. One can see even lower serum sodium concentrations in patients with a clear sensorial. Volume deprivation leads to renal wasting of sodium (E) is plain wrong. What the kidney does when there is volume depletion is to increase reabsorption of sodium, not to dump it. -
Question 27 of 100
27. Question
1 pointsA 73 year old female with diabetes, hypertension is admitted after sustaining a fall. She is found to have a right femoral neck fracture and is taken for an open reduction and internal fixation. Her post-op course is uneventful, with physical therapy beginning on post-op day 2. On day 3 she complains of pain in the hip extending to the ankle. There is increased distal edema and marked calf tenderness. The next step in the management is
Correct
Incorrect
Explanation:
The most likely diagnosis is deep venous thrombosis (DVT), for which this patient is high risk given her bed rest secondary to the orthopedic operation. She should have been on appropriate DVT prophylaxis, ideally with low molecular weight heparin. Although the question stem is certainly suggestive of this diagnosis, with the appropriate imaging, therapy should not be initiated, especially in an elderly postoperative patient. Although a venogram (E) would be the gold standard for the diagnosis, it is highly invasive; therefore, the appropriate diagnostic modality is a duplex venous flow study, as it is entirely non-invasive and has sensitivity and specifically greater than 90%.
Therapy would then entail systemic anticoagulation with either heparin or low molecular-weight heparin (C) followed by warfarin (D). It is crucial that warfarin not be started before adequate anticoagulation with a heparin product because of the well described initial pro-coagulopathy state after initiating warfare therapy due to its inhibition of proteins C and S. Administering IV furosemide (A) is based on this patient´s distracting cardiac history, making congestive heart failure (CHF) exacerbation a possibility; but there is no mention of any other physical exam findings consistent with this diagnosis, and the lower-extremity edema is unilateral, whereas congestive heart failure would present as bilateral edema. If CHF exacerbation were in fact the diagnosis, the administration furosemide would certainly be appropriate. -
Question 28 of 100
28. Question
1 pointsA 67 year old obese male is diagnosed with type 2 diabetes mellitus at a screening conducted at a local shopping center and is then referred for evaluation and several abnormalities are uncovered. He has an alkaline phosphatase 4 times the upper limit of normal, total billrubin is 2.7 mg/L, his Hb is 9 mg/L, and stools are positive for occult blood. Upper abdomen sonogram shows dilated intra-hepatic and extra hepatic ducts. Which one is the appropriate next diagnostic test?
Correct
Incorrect
Explanation:
The man obviously has early obstruction of his billiary tract, of which he had not been clinically aware. He also has blood in the lumen of the gut, and has been bleeding sufficiently to become anemic. The only single lesion that could account for both of these findings is a cancer of the ampulla of Vater. Endoscopic examination should reveal it and allow biopsies to confirm the diagnosis. CT scan of the abdomen (A) can reveal sources of billiary obstruction if they are large (cancer of the head of the pancreas) or calcified (gallstones), but is unlikely to show a small ampullary cancer. Colonoscopy (B) would have been the thing to do if all he had was anemia and blood in the lumen of the gut, in which case the diagnostic suspicion would have been cancer of the right colon. But cancer of the colon could not account for the billiary obstruction. Percutaneous cholangiogram (D) is sometimes done to visualize the billiary tract but ERCP is usually preferred. In any event, we should be able to establish the diagnosis in this case by just doing the “E” of the ERCP (endoscopy). Upper gastrointestinal series (E) is a poor second choice to direct visualization when trying to confirm the presence of an ampullary cancer.
-
Question 29 of 100
29. Question
1 pointsA 58 year old man with ulcerative colitis for 30 years and sclerosing cholangitis for past 10 years has weight loss, jaundice, pruritus, and progressive right upper quadrant abdominal pain. Exam shows hepatomegaly. His bilirubin level is 16 mg/L with 13 mg/L direct, minimally elevated transaminases, and ALP level 10 times the normal. Abdominal CT shows ducal dilation in hepatic lobes, small gall bladder and inability to visualize the right and left hepatic ducts. His tumor is most likely located in
Correct
Incorrect
Explanation:
The laboratory picture is highly suggestive of obstructive jaundice. Patients who have sclerosing cholangitis are at high risk for developing cholangiocarcinoma (lifetime risk is estimated to be 10 to 15%). Sonogram is the best initial study, but it is often difficult to interpret. In these cases, a CT scan is expected to provide greater detail and might show the tumor. Based on the results, a good guess as to the anatomic location of the tumor can be made. Klatskin tumors are a subtype of cholangiocarcinoma that occur at the confluence of the right and left hepatic bile ducts with or without a thickened wall. This results in intrahepatic ducal dilation and a contracted gall bladder because of the obstruction to outward biliary flow. Obstruction to outward flow also leads to a predominantly conjugated hyperbilirubinema. Cholangiocarcinoma occurring at the ampulla of Vater (A) or the common bile duct (B) or a tumor in the head of the pancreas (E) large enough to compress the distal billiary tree would be expected to produce a distended gall bladder and dilation of the intrahepatic and extra hepatic billiary ducts proximal to the tumor. A distended gall bladder without intra or extra hepatic billiary duct dilation is suggestive of a cystic duct tumor (D) or stone.
-
Question 30 of 100
30. Question
1 pointsA 63 year old female has a 4 cm, hard mass under the nipple and areola of her left breast. There is no dimpling or ulceration of the skin over the mass. Core biopsy has established a diagnosis of infiltrating ductal carcinoma, and a mammogram shows no other lesions. CXR and LFTs are normal. She has no symptoms suggestive of brain or bone metastasis. Which treatment should be offered?
Correct
Incorrect
Explanation:
Any malignant mass >4 cm in diameter exceeds the criteria to perform a lumpectomy; therefore, mastectomy is indicated. However, mastectomy alone is not sufficient. Axillary node sampling is required in all suspicious lesions in order to determine postoperative systemic therapy.
Lumpectomy alone (A) cannot be done because the tumor is >4 cm in diameter.
Lumpectomy, axillary sampling and postoperative radiation (B) would have been the correct answer for a tumor 4 cm. Total mastectomy alone (C) would have been a wise palliative choice if the patient had evidence of distant metastasis.
Radical mastectomy (E) is no longer used for the treatment of breast cancer. It offers no survival advantage over the less aggressive modified radical mastectomy. -
Question 31 of 100
31. Question
1 pointsA 76 year old man with chronic bronchitis develops a bulge in his left groin. On examination, an elongated swelling is seen above the medial end of the inguinal ligament. When the patient coughs, the swelling enlarges, but does not descend into the scrotum. Which one of the following is the most likely diagnosis?
Correct
Incorrect
Explanation:
An inguinal hernia usually produces a painless bulge in the groin or scrotum. If the hernia can slide back and forth, the bulge may enlarge when a man stands or strains. The bulge may get smaller or disappear when the man lies down. If the hernia is incarcerated, the bulge does not get smaller or disappear. Strangulated hernias usually become very painful within minutes or hours.
To diagnose an inguinal hernia, a doctor examines the groin and the area around it. For the examination, a man may have to stand and cough or strain. Coughing or straining produces pressure in the abdomen and makes a hernia more obvious. To determine whether the hernia is reducible or incarcerated, the doctor usually tries to push the loop of intestine back into the abdomen by pushing on the upper part of the scrotum with a gloved finger. -
Question 32 of 100
32. Question
1 pointsA man aged 25 presents with a history of recent bilious vomiting followed by a large volume of bright red hematemesis. He is pale, sweaty and anxious and complains of mild epigastric pain. Blood pressure is 80/50 mm Hg, pulse 128/minute. All of the following steps are correct, except
Correct
Incorrect
Explanation:
Esophaegal varices are dilated veins in the distal esophagus or proximal stomach caused by elevated pressure in the portal venous system, typically from cirrhosis. They may bleed massively but cause no other symptoms. Diagnosis is by upper endoscopy.
Treatment involves fluid resuscitation, endoscopic banding (sclerotherapy second choice) IV octreotide and possibly a transjugular intrahepatic portosystemic shunting (TIPS) procedure.
If bleeding continues or recurs despite these measures, emergency techniques to shunt blood from the portal system to the vena cava can lower portal pressure and diminish bleeding. A TIPS procedure is the emergency intervention of choice. TIPS is an invasive radiologic procedure in which a guidewire is passed from the vena cava through the liver parenchyma into the portal circulation.
The resultant passage is dilated by a balloon catheter, and a metallic stent is inserted, creating a bypass between the portal and hepatic venous circulations. Stent size is crucial. If the stent is too large, hepatic encephalopathy results because of diversion of too much portal blood flow from the liver. If the stent is too small, it is more likely to occlude. -
Question 33 of 100
33. Question
1 pointsAll of the following are not true regarding testicular torsion except
Correct
Incorrect
Explanation:
Testicular torsion is the most common cause of spontaneous testicular pain in boys age 12 years and older and is uncommon before 10 years of age. There is inadequate fixation of the testis within the scrotum, resulting in excessive mobility. Shortly after torsion, there is acute pain and swelling of the scrotum, which is very tender. The normal cremasteric reflex is nearly always absent. The differential diagnosis includes strangulated inguinal hernia, which most often causes inguinal swelling, unlike torsion. There is no antecedent association of hydrocele with testicular torsion. Spermatogenesis can be lost within 6 hours of the loss of blood flow, making prompt diagnosis and surgical detorsion and fixation essential. Either a Technetium-99m pertechnetate flow scan of color Doppler Ultrasonography is 95 % accurate in determining flow, but results can be misleading if torsion is less that 360. These tests are reasonable, but should not delay surgical intervention given the small window of therapeutic opportunity.
-
Question 34 of 100
34. Question
1 pointsA female aged 18 years is admitted unconscious to the Emergency Department following a car accident. She has facial lacerations and a displaced tibial fracture. What is of first priority in her management?
Correct
Incorrect
Explanation:
ABC is a mnemonic for memorizing essential steps in dealing with an unconscious or unresponsive patient. It stands for Airway, Breathing and Circulation. If the patient´s airway is blocked, oxygen cannot reach the lungs and so cannot be transported round the body in the blood. Ensuring a clear airway is the first step in treating any patient. Common problems with the airway involve blockage by the tongue or vomit. Initial opening of the airway is often achieved by a “head tilt, chin lift” or jaw thrust technique, although further maneuvers such as intubation may be necessary. The patient is next assessed for breathing. Common findings during an assessment of breathing may include normal breathing, noisy breathing, gasping or coughing. The rescuer proceeds to act on these based on his/her training. Generally at this point it will become clear whether or not the casualty needs supportive care (such as the recovery position) or rescue breathing. Once oxygen can be delivered to the lungs by a clear airway and efficient breathing, there needs to be a circulation to deliver it to the rest of the body. This can be assessed in a number of ways, including a pulse check, ECG analysis, or capillary refill time.
-
Question 35 of 100
35. Question
1 pointsA patient develops both an indirect and direct hernia. This is known as a
Correct
Incorrect
Explanation:
A hernia is an abnormal protrusion of tissue through a fascial defect. A Spigelian hernia is an acquired hernia through the linea semilunaris (the lateral border of the rectus abdominus muscle). A Richter hernia occurs when part of the bowel becomes incarcerated in the defect. A direct hernia occurs because of a defect in the transversalis fascia in Hesselbach´s triangle, as opposed to an indirect inguinal hernia, in which the processus vaginalis falls to be obliterated. Littre´s hernia contains a Meckel´s diverticulum within it. A pantaloon hernia is a combined direct and indirect hernia.
-
Question 36 of 100
36. Question
1 pointsA man has a sigmoid colon resection for diverticulitis. Gentamicin and ampicillin are given for one week. One week later, the man complains of fever (39.5°C) and abdominal pain. The infection is probably caused by which one of the following organisms?
Correct
Incorrect
Explanation:
B. fragilis is a common bowel organism and will not be covered by either gentamicin or ampicillin. E. coli and P. mirabilis are Gram-negative rods and will typically be covered by gentamicin. C. perfringes and S. Aureus are not bowel flora. -
Question 37 of 100
37. Question
1 pointsA 54 year old woman presents with erythematous papules in the region of surgical scar after two years of right modified mastectomy for breast cancer. These papules are most likely due to which of the following?
Correct
Incorrect
Explanation:
Recurrent breast cancer is likely to produce small nodules in the scar tissue. Streptococcal infection is most likely to involve laryngeal, pharyngeal, or tonsillar tissue. Staphylococcal infections are most likely to occur within a few weeks after surgery. Candida Albicans prefers deeper locations where heat, moisture, and macerated tissue are available to promote growth. Erysipelas lesions are more likely to appear as vesicles than papules.
-
Question 38 of 100
38. Question
1 pointsAll of the following have been described in acute appendicitis, EXCEPT
Correct
Incorrect
Explanation:
If appendicitis evolves slowly adjacent organs such as the terminal ileum, cecum, and omentum may wall off the appendiceal area, resulting in an abscess. Although obstruction of the appendiceal lumen with a fecalith is thought to be a primary initiating event in acute appendicitis, it cannot be identified in every case. Alternatively, mucosal ulceration has been demonstrated as another initiating event. Following obstruction of the lumen, mucous secretion is impaired, resulting in distention of the organ. Luminal bacteria multiply and invade the appendiceal wall as its veins become engorged and its arterial supply is compromised.
-
Question 39 of 100
39. Question
1 pointsIn which of the following conditions is splenectomy always indicated?
Correct
Incorrect
Explanation:
Splenectomy is always indicated in two conditions that result in primary hypersplenism: heriditary spherocytosis and primary splenic tumor. It may be indicated in causes of secondary splenism when the primary cause of illness cannot be cured. It is never indicated in hypersplenism secondary to infection, immune mediated reaction, or agranulocytosis. The treatment of portal hypertension is portal decompression, although splenectomy may rarely be indicated.
-
Question 40 of 100
40. Question
1 pointsA 66 year old woman presents with pain in her lower calves which is worse in warm weather. She has varicose veins on examination. Which of the following statement is FALSE regarding varicose veins?
Correct
Incorrect
Explanation:
Initial treatment of varicose veins is noninvasive compression stockings, weight control, elevation of legs above the head, avoidance of high heels, and whirlpool baths. If conservative treatment fails after three to six months or if complications such as phlebitis develop, surgery or sclerotherapy would be the treatment of choice. All of the remaining statements regarding varicose veins are true.
-
Question 41 of 100
41. Question
1 pointsA 100 pound block falls from a height on a 35 year old man at work and cause a severe injury. On his arrival in the emergency room, he complains of pain in his left shoulder and he has some abdominal distention. Which of the following is a major concern in this patient?
Correct
Incorrect
Explanation:
Left shoulder pain and abdominal distention (due to hemoperitoneum) are findings in splenic rupture, due to the distribution of the nerve endings supplying the spleen. Cardiac tamponade would present as an acute cardiovascular emergency; intussusception is not associated with acute trauma; a humeral fracture would not cause abdominal distention; and fracture of the thoracic spine also would not lead to abdominal distention.
-
Question 42 of 100
42. Question
1 pointsIn which of the following condition lymphadenopathy may NOT occur?
Correct
Incorrect
Explanation:
In adults, inguinal lymph nodes are often palpable and are less than two centimeters in size. Lymph node enlargement may result from an increase in the number of benign lymphocytes and macrophages in response to antigenic stimulation. Virtually any disease characterized by immune cell activation, such as systemic lupus erythematosis, rheumatoid arthritis, serum sickness, or angioimmunoblastic lymphadenopathy, will produce lymphadenopathy. In amyloidosis, deposition of fibrillar material in lymph nodes results in lymphadenopathy. Infiltration of lymph nodes by inflammatory cells or hyperplasia of the lymphoid or phagocytic cells produces lymphadenopathy in infectious diseases. In situ proliferation of malignant lymphocytes or macrophages, such as occurs in lymphomas, Hodgkin´s disease, or leukemias, also results in enlargement of lymph nodes.
-
Question 43 of 100
43. Question
1 pointsAll of the following are useful in the management of acute diverticulitis EXCEPT
Correct
Incorrect
Explanation:
Non-absorbable oral antibiotics are of no use in the management of diverticulitis. IV broad spectrum antibiotics are definitely indicated. Patients should be kept from eating or drinking (A), started on IV fluids (C), and pain should be controlled with narcotics (D) if necessary Morphine should be avoided, as it increases colonic contraction. Patients should undergo a barium enema (E) one week after the attack.
-
Question 44 of 100
44. Question
1 pointsA 56 year old man is diagnosed with gastric carcinoma. Staging indicates that there is distant spread of the disease. What should be the appropriate management?
Correct
Incorrect
Explanation:
The survival rate for patients with distant spread of gastric carcinoma is poor. Patients with distant spread should receive palliation treatment only for such conditions as obstruction. Radiation and chemotherapy are not effective. Subtotal and total gastrectomy are indicated for localized tumors.
-
Question 45 of 100
45. Question
1 pointsWhen should the mammograms be performed?
Correct
Incorrect
Explanation:
Mammograms alone should not serve as the only screen for the development of breast cancer. Women should be instructed in, and encouraged to perform, breast self examination, since this can identify a lesion before the annual or biannual mammography exam. Breast ultrasonography should not be a routine exam in addition to mammography, but should only be perfumed to demonstrate cystic changes when a suspicious lesion is found at a mammogram. Mammography is not advised in most women before the age of 40, unless they have detected lumps or there is another compelling reason for earlier studies. The amount of radiation in a mammogram is too high to justify using it routinely in all women beginning at the onset of puberty. Mammography can detect lesions that are not palpable and, thus, should not be reserved only for those with lumps.
-
Question 46 of 100
46. Question
1 pointsA 39-year-old is shot once with a 38 caliber revolver. The entry wound is in the left midclavicular line, 2 inches below the nipple. There is no exit wound. He is hemodynamically stable. A chest x-ray film shows a small pneumothorax on the left and demonstrates the bullet to be lodged in the left paraspinal muscles. In addition to the appropriate treatment for the pneumothorax which of the following will this patient most likely need?
Correct
Incorrect
Explanation:
Although this vignette describes a gunshot wound of the chest, we must remember that the chest and the abdomen are not simply stacked upon one another. There is a dome-shaped diaphragm that separates them, and thus an area where chest and abdomen overlap. Any gunshot wound below the nipples and above the pubic symphysis is considered to involve the abdomen, and such is the case here. The management of all gunshot wounds of the abdomen requires exploratory laparotomy.
Barium swallow and bronchoscopy are indicated if there are signs suggestive of injury to the esophagus and the tracheo- bronchial tree, respectively, (coughing up blood, spitting up blood), or if the anatomic trajectory of the bullet puts the track in their vicinity, making an injury to these organs likely. Here, we have an entry wound on the left and a bullet lodged on the left: the midline has not been crossed.
CT scan would not change the surgical approach and the surgical indication. CT scan is called for in cases of blunt trauma to diagnose intraabdominal bleeding and to identity intraabdominal injuries.
Taking out the bullet is unnecessary if the missile is not pressing on some vital structure. A bullet embedded in a muscle can be left there. -
Question 47 of 100
47. Question
1 pointsA 57 year old diabetic male is admitted with a diagnosis of epididymo-orchitis. WBC count was 22,000/mL He was started on IV ciprofloxacin, scrotal elevation, and scrotal ice packs. On third day there is persistent right hemi scrotum edema with scrotal wall erythema. The testicle is tender to palpation. Blood and urine cultures are negative. Ultrasound shows an increase in regional blood flow of the right epididymis and confirms the thickened edematous scrotal wall and it also shows complex, loculated fluid collection within the right hemi scrotum. The most appropriate management at this time is
Correct
Incorrect
Explanation:
This patient has an epididymo-orchitis that is complicated by the development of a scrotal abscess. The appropriate treatment of an abscess anywhere in the body is drainage. The technique of drainage can vary, depending on the location of the abscess with a scrotal abscess, exploration and open drainage is the most appropriate. The location is easy to gain access to, the actual procedure is generally well tolerated, and this is the only technique that ensures all the purulent fluid is drained completely. Aspiration (A) carries the risk of not removing all the fluid; therefore, there is a risk for recurrence. Aspiration of an abscess should be reserved for collections that would require major surgical intervention to reach. A change in this patient´s antibiotics will not treat him completely (B). Management of a patient with presumptive epididymo-orchitis on admission with intravenous antibiotics is appropriate. Most patients with this diagnosis improve with antibiotics when a patient´s condition does not respond appropriately, however, it is necessary to investigate other possible diagnoses. As stated, this patient requires surgical drainage.
This patient has not improved from a clinical or laboratory standpoint. His ultrasound shows an abscess; therefore, continuing with the current regimen would be inappropriate (C). Fournier gangrene (D) is defined as a necrotizing fasciitis that presents as a progressive, fulminating infection of the male genitalia, initially, the patient may present with only induration of the penis or scrotal skin. Erythema and crepitus develops and quickly extends over the perineum and abdominal wall. Treatment requires immediate wide surgical debridement of the involved skin and subcutaneous tissue. -
Question 48 of 100
48. Question
1 pointsA 46 year old female has bone pain, myalgias, constipation, and a depressed mood. She has been hospitalized twice in the last year for urolithiasis. Labs reveal hypercalcemia and elevated PTH level. Her renal function is normal, and neck ultrasound reveals a parathyroid mass. Which laboratory abnormality would be expected in this patient?
Correct
Incorrect
Explanation:
The patient described is suffering from primary hyperthyroidism. The most common cause of primary hyperthyroidism is a parathyroid adenoma; parathyroid carcinoma and hyperplasia are much less common. This condition most commonly occurs in women over the age of 40 years as a solitary adenoma. Genetic syndromes such as MEN I, IIa, and IIb may also be associated with parathyroid adenomas, among other findings. Typical presenting complaints are those associated with hypercalcemia. These include recurrent urolithiasis, constipation, myalgias, bone pain, polyuria, fatigue, and psychologic changes. Patients may be asymptomatic at presentation, however, with the diagnosis made following discovery of an elevated calcium level on routine blood tests. Laboratory studies typically reveal an inappropriately elevated parathyroid hormone (PTH) level, hypercalcemia, hypophosphatemia, and elevated urine calcium. Chronically elevated PTH levels can cause defective bone mineralization and remodeling, leading to osteitis fibrosa cystica, a condition characterized by the formation of “brown tumors” in bone, particularly in the jaw. In cases in which the bone is significantly affected, the serum alkaline phosphatase is elevated, not decreased (E). Hypernatremia (A) typically occurs in the setting of excessive free water loss and is not associated with primary hyperparathyroidism. Hyperphosphatemia (B) and low levels of dihydroxy (24, 25(OH) 2) vitamin D occur in the setting of renal failure/renal osteodystrophy, one cause of secondary hyperparathyroidism. Hyponatremia (C) can occur in the syndrome of inappropriate antidiuretic hormone (SIADH), which is a paraneoplastic condition associated with small cell carcinoma of the lung. Another paraneoplastic syndrome associated with this malignancy is hypercalcemia caused by production of PTH related peptide by the malignant cells, resulting in hypercalcemia and low PTH levels. -
Question 49 of 100
49. Question
1 pointsA 57 year old man undergoes a liver transplantation that goes well, without any noted complications. 10 days later his levels of gamma glutamyl transferase (GGT), alkaline phosphatase, and billrubin began to rise. The most appropriate next step in diagnosis is which one of the following?
Correct
Incorrect
Explanation:
In liver transplants, technical problems with the biliary and vascular anastomosis are the most common cause of early functional deterioration, and are therefore, the first anomalies to be sought. Antigenic reactions are less common in liver transplants than in other solid organ transplants, in which deterioration of function after 10 days would suggest an acute rejection episode, and appropriate biopsies would be needed to confirm the diagnosis. Options A, B, and C are centered on liver biopsy, which would be done only after technical problems with the biliary and vascular anastomosis have been ruled out. Preformed antibodies (D) are responsible for hyperacute rejection, which would be evident within minutes of establishing blood flow to the graft.
-
Question 50 of 100
50. Question
1 pointsAn 8 year old boy has had 2 days of abdominal pain that caused him to miss school, and he woke up this morning with bloody diarrhea. He is exquisitely tender in the right lower quadrant and the suprapubic region. He has fever of 100.9°F and a WBC count of 13,000/mm3. He is taken for an appendectomy. In the operating room, appendix is normal but a Meckel diverticulum that is grossly inflamed. The accurate statement regarding his condition is
Correct
Incorrect
Explanation:
This patient presents with a classic case of Meckel diverticulitis, an inflammation of the remains of the embryonic omphalomesenteric duct that is usually located approximately 50 cm from the ileocecal valve. The “rule of 2´s” refers to:
- the location within 2 feet of the ileocecal valve 4 its presence in 2% of the population
- That it is symptomatic in only 2% of those in which it occurs
- That it consists of two types of mucus (usually intestinal and gastric)
Most of the time, bleeding occurs without warning and stops spontaneously.
In asymptomatic patients, or those presenting without complications diverticulectomy alone is usually enough (A) However, cases that present with complications, such as bleeding, perforation, obstruction, and strangulation, need to be treated with resection of the diverticulum and the adjacent bowel segment (B), which is curative (D). Bowel rest and intravenous antibiotics (C) are part of the conservative management of this condition; however, this is not likely to be enough treatment for patients presenting with bleeding and other complications
Technetium-99m pertechnetate scintigraphy, or Meckel scan, is an important diagnostic modality in which the insertion of a radioisotope with high affinity for the gastric mucosa (not colonic mucosa, E), allows for visualization of the ectopic tissue in the ectopic diverticulum, which is likely producing the mucosal erosions and the bleeding. -
Question 51 of 100
51. Question
1 pointsA 77 year old man is brought with altered mental status. His past history is significant for hypertension, osteoarthritis, and frequent falls. BP is 100/50 mm Hg and temperature is 38.5°C. On CNS exam he is oriented only to person and has delayed tendon reflexes. CBC shows leukocytosis and urine culture is positive for E. coli. IV ciprofloxacin is started and his WBC count improves, but his mental status does not improve. The next diagnostic step is
Correct
Incorrect
Explanation:
The question points out that resolving urosepsis did not affect this man´s mental status, implying another etiology. His history of frequent falls is a predisposing factor for head trauma, and previous or ongoing cerebrovascular ischemia or hemorrhage must be ruled out with a CT scan of the head. A recent change in mental status could be due to more common causes that need to be ruled out before proceeding with more expensive testing; these include hypoxemia, hypoglycemia, electrolyte abnormalities, toxins and polypharmacy fever, psychiatric disorders, brain infections, and brain injury such as an acute or chronic subdural herntoma, which is frequently found in patients with a history of multiple falls. The etiology of this patient´s altered mental status should be explored outside the genitourinary tract, based on the specific mention that his urinary tract infection is improved. This makes renal ultrasound (D) unlikely to reveal further detail on the cause of the patient has altered mental status. Duplex ultrasound of the leg (B) in order to rule out deep venous thrombosis is unlikely to reveal the real cause of this patient has altered mental status. There are no signs or symptoms related to deep venous thrombosis or pulmonary embolism that may suggest the need for this diagnostic procedure. Stopping all medications, (C) is not the appropriate step in this scenario. Even though polypharmacy is one of the common causes of altered mental status in geriatric patients, this man is on few medications; stopping them is more likely to harm him than to alleviate his current problem. Water restriction (E) is likely to dehydrate this patient and further increase his serum sodium level, which is normal at the moment and is an unlikely cause of his altered mental status. Therefore, this is not the appropriate step to be taken at the moment.
-
Question 52 of 100
52. Question
1 pointsA 59 year old man is brought because he has 12 weeks of worsening pain in his right shoulder and left arm, and has had difficulty urinating for the past 10 months. Muscle strength is diminished in both lower limbs. Temperature and pain sensation are absent in the right and left foot. Proprioception is spared. Plantar and deep tendon reflexes at the knees and ankles are absent. Prostate is large, firm, and non-tender on rectal exam. The diagnosis is
Correct
Incorrect
Explanation:
Carcinoma of the prostate is the most common malignancy in men in the U.S., and is the second most common cause of cancer death in men older than 55. The disease is more common among blacks than whites. Most carcinomas of the prostate are slow growing, and greater than 80% are stage C or D at diagnosis. Symptomatic patients usually complain of difficultly voiding end, with advanced disease, have symptoms of spinal compression. Hematogenous metastases occur to the bone more frequently than to the viscera, and in enlarged, non-tender prostate are palpable on rectal exam. Amyotrophic lateral sclerosis (A) is progressive motor neuron disease that affects both upper and lower motor neurons. Patients present with a gradual onset of asymmetric weakness of the distal limb. Even in advanced disease, sensory and bladder function are preserved. On exam, there is hyperactivity of muscle stretch reflexes. Pot disease (C) is vertebral osteomyelitis primarily involving the thoracic spine. Patients usually complain of neck or back pain. Palpation of the involved area results in pain, and physical exam reveals spasm of par spinal muscles and limitation of motion. Sciatica (D) is caused by irritation of the sciatic nerve (L4, L5, S1). Pain originating in this nerve radiates down the posterior aspect of the thigh and the posterior and later aspect of the leg. Sciatica generally stops at the ankle and is associated with paraesthesias (tingling and numbness) that extend distally into the foot. Reflex loss, muscle atrophy and weakness, and fascicular twitching may be present on physical exam. Spinal stenosis (E) which typically affects elderly patients is a narrowing of the lateral recess and inter vertebral foramen with resultant encroachment on the roots of the cauda equina. The pain is bilateral, worsens with standing and walking, and is relieved by rest. -
Question 53 of 100
53. Question
1 pointsA 44 year old man develops fever, diarrhea and non-productive cough two weeks after receiving a bone marrow transplant. CXR shows bilateral lung infiltrates. He dies of overwhelming sepsis and multiorgan failure. Autopsy reveals CMX and extensive cell necrosis in the intestinal epithelium and skin. This complication of bone marrow transplantation is principally mediated by
Correct
Incorrect
Explanation:
Allergenic bone marrow transplantation has become a frequent therapeutic approach to a variety of conditions, including leukemic diseases. The patient undergoing bone marrow transplantation is profoundly immunosuppressed and prone to developing opportunistic infections. The clinical picture described in this case is consistent with graft versus host disease (GVHD), in which T cells (both helper and suppressor cells) of the engrafted marrow react against the recipient´s antigens, thus triggering inflammation and injury to the host tissues. The most severely affected organs include the immune system, gastrointestinal tract liver, skin, and lungs. This complication may be acute (this case) or chronic. CMV pneumonia is a frequent fatal complication in the acute stage. The chronic stage is characterized by progressive fibrosis of affected organs.
B-lymphocytes of a bone marrow grail (A) do not play a significant role in GVHD. Leukemic cells (B) may give rise to recurrence of the original disease, which must be distinguished from GVHD. The combination of skin rash and opportunistic infections strongly favor GVHD. In addition, single cell necrosis in the epithelia of skin, GI tract and liver is highly characteristic of GVHD.
Natural killer cells of the recipient (C) and T-lymphocytes of the recipient (E) play a crucial role in mediating rejection of allergenic marrow transplants. -
Question 54 of 100
54. Question
1 pointsA 60 year old man has erectile dysfunction. Besides diabetes, his medical history is significant for COPD and mild pulmonary hypertension, for which he is taking a vasodilator that has greater effect on the venous system than on the arterial system, albuterol, and ipratropium inhalers. Which therapy is contraindicated in him?
Correct
Incorrect
Explanation:
All of the choices listed are used in the treatment of organic erectile dysfunction. However, to answer this question you must first recognize that this patient is being treated with nitroglycerin for his pulmonary hypertension secondary to his COPD. Sildenafil citrate is contraindicated in patients who use nitrate containing medication. Sildenafil citrate works by inhibiting the type 5 cGMP phosphodiesterase enzyme, which is primarily located in the corpus cavernous smooth muscle. This inhibition leads to an increase in cGMP, which causes a decrease in intracellular calcium and ultimately relaxation of cavernously smooth muscle. This results in erection. Nitroglycerine is converted to nitric oxide (NO) in the blood through a mechanism that is poorly understood. NO is a potent vasodilator; its effect on veins is more than that on arteries, thus leading to pooling of blood in the lower extremities and causing orthostatic hypotension secondary to the reduced preload. Sildenafil citrate differs from the other medications listed in that it will not generate an erection by itself; therefore, stimulation must occur. Because of its mechanism of action, it potentiates the effect of nitrate containing medications that may cause life threatening hypotension. Sildenafil has also been used in patients with primary pulmonary hypertension; however, it has not been shown to have satisfactory results in secondary pulmonary hypertension, which is the case in this question.
Epoprostenol (A) is an analogue or prostacyclin (PGI 2) which is an eicosanoid that primarily prevents platelet aggregation and clumping in the clotting cascade. PGI2 also causes vasodilatation and is administered as a continuous infusion that requires a semi-permanent central line catheter. Bosentan (B) is an endothelin receptor antagonist. Endothelin is a very potent vasoconstrictor that binds to smooth muscle endothelial receptors. Bosentan blocks these receptors, producing vasodilatation. Combination therapy with nitroglycerin has been approved for refractory cases of pulmonary hypertension and has shown a decrease in mortality. A penile prosthesis (D) is typically the final treatment modally offered to patients suffering from impotence. It is invasive, requires anesthesia, and because it is a prosthetic implant carries the risk for infection and malfunction. All of these factors increase the morbidity of the patient. This patient not a good candidate for a surgical procedure, primarily due to his COPD and pulmonary hypertension. Once all other options have been exhausted, then a penile prosthesis may be offered to this patient. Vacuum erection devices (E) work by placing a cylinder over the penis and having a vacuum pump draw blood into the penis. An occlusive band is placed around the base of the penis to maintain the erection. These devices are contraindicated in patients with a predisposition to priapism, significant angulations or fibrosis of the penis, penile implant, bleeding disorder, or on anticoagulation medications. -
Question 55 of 100
55. Question
1 pointsA 63 year old chronic alcoholic and diabetic farmer has chronic right sided otitis media and difficulty swallowing. Multiple courses of antibiotics have failed to resolve his ear infection. Exam reveals a posterior cervical mass on lymph node palpation, and oral mucosa exam reveals an ulcerated mass in the right posterior nasopharynx, biopsy of which reveals squamous cell carcinoma. Which one likely contributed to the development of his disease?
Correct
Incorrect
Explanation:
The patient described has a squamous cell carcinoma (SCC) of the pharynx. SCC of the head and neck has been associated with alcoholism, chronic tobacco use, and infection with the human papilloma virus (HPV), particularly monogenic HPV strains such as type 16. Oropharyngeal SCC most commonly affects men over the age of 50 years, though men may be more likely to abuse alcohol and tobacco, and the most commonly involved sites are the palate and tongue. Lesions classically begin as red or white plaques (erythroplakia and leukoplakia, respectively), but tumors in the nasopharynx and or pharynx often do not become clinically evident until disease is severe (stage III to IV). Lesions of the nasopharynx may occlude the Eustachian tube, causing accumulation of mucus in the middle ear, which is then predisposed to repeated, chronic infections. Aflatoxin (A) is produced by some species of Aspergillus and typically occurs in locations where crop grains are stored. It is associated with the development of hepatocellular carcinoma. Arsenic (C) exposure is associated with squamous cell carcinoma of the skin, but it does not cause oropharyngeal SCC. Obesity (E) is associated with a greater risk for diabetes, stroke, and coronary artery disease and has a detrimental effect on expected lifespan, but it is not directly associated with any particular malignancies.
-
Question 56 of 100
56. Question
1 pointsA 54 year old woman has a 3 day history of chills, spiking fever, and right sided abdominal pain. She underwent laparoscopic cholecystectomy 3 months ago and was told that the operation had gone well. 10 days later she developed jaundice and required a second operation, an open laparotomy, but she does not know what was done. Today she is jaundiced, and labs show total bilirubin of 8 mg/L, indirect bilirubin of 1 mg/L, ALT 56 mg/L, AST 52 mg/L and ALP 1052 mg/L. Sonogram shows dilated intrahepatic ducts but common duct could not be visualized. The most likely diagnosis is
Correct
Incorrect
Explanation:
The story is classic for an iatrogenic stricture of the common bile duct (CBD). It presents with biliary colic and obstructive jaundice (very high alkaline phosphates, direct hyperbilirubinemia). This CBD stricture got infected and has led to cholangitis due to partial or complete obstruction of the CBD, with increased intra-luminal pressures, bacterial infection of the bile with multiplication of the organisms within the duct, and seeding of the bloodstream with bacteria or end toxin. Cholangitis can rapidly become life threatening condition. Clinical presentation of cholangitis is the Charcot triad of fever and chills, jaundice, and right upper quadrant abdominal pain in most patients. Hepatic abscess (A) would have been seen in the sonogram, and would not have produced dilated intrahepatic ducts. Hepatitis (B) would show high transaminases and minimally elevated alkaline phosphatase. The ducts would have been of normal size in the sonogram. Retained stones (D) are a possibility, but it is far less likely than stricture. Had the second surgery been done to retrieve stones and some left behind, the obstruction would typically be near the ampulla, and the common duct would have been dilated in the sonogram. Subphrenic abscess (E) would produce fever, but would not change liver function tests or affect the size of the intrahepatic ducts. Furthermore, the sonogram would have easily detected the abscess.
-
Question 57 of 100
57. Question
1 pointsA 30 year old woman sustains multiple injuries including upper and lower extremity fractures following an accident. She is awake and alert and remembers hitting her abdomen against the steering wheel. Exam shows a rigid and tender abdomen. There is severe tenderness when external pressure is applied to her abdomen and then suddenly released. Bowel sounds are absent. Appropriate step in evaluating potential intra-abdominal injuries is
Correct
Incorrect
Explanation:
The presence of an “acute abdomen,” which this woman has, is an indication for exploratory surgery, and prompt repair of the injuries (probably affecting hollow viscera that have produced the signs of peritoneal irritation. Continued clinical observation (A) would be irresponsible when it is clinically obvious that she already has an acute abdomen. CT scan (B) is ideal when the issue is potential intra abdominal bleeding in a hermodynamically stable patient who can be safely sent to the radiology department. CT scan might even be a good idea if the picture of acute abdomen were equivocal. However it is not needed here. Diagnostic peritoneal lavage (D) or sonograms done in the emergency department (C) are our options when we suspect intra abdominal bleeding and the patient is too unstable to be sent anywhere. As pointed out above, however, when an acute abdomen has clearly developed, it is time to operate.
-
Question 58 of 100
58. Question
1 pointsA 52 year old chronic smoker and drinker presents with weight loss and difficulty eating. He noticed some difficulty swallowing his dinner over the past 2 months. Initially he had trouble with roasted potatoes, but now even the mashed potatoes are not “going down smoothly.” He has also lost 15 lbs since his last visit. On exam he appears weak. No lymph nodes are palpable. His abdomen is soft, non-tender. The likely diagnosis is
Correct
Incorrect
Explanation:
Cancer of the esophagus shows the classic progression of dysphagia starting with meat then other solid foods, eventually liquids, and finally saliva. Significant weight loss is also seen. Smoking and drinking are risk factors for squamous cell carcinoma, and blacks are affected more commonly than whites. Achalasia (A) is seen more commonly in women. There is dysphagia also, but it is worse for liquids. There is occasional regurgitation of undigested food. Colon cancer (B) does not present with swallowing problems, although guaiac positive stools are present. If the cancer is present on the left side, there may be bloody bowel movements. Adenocarcinoma of the esophagus (C) is seen in people who have longstanding gastroesophageal reflux. Carcinoma of the stomach (D) is usually seen in elderly patients. There is anorexia and weight loss; however, there is vague epigastria distress and early satiety, rather than dysphasia. Hematemesis is possible.
-
Question 59 of 100
59. Question
1 pointsA 31 year old HIV positive man is brought to the ER complaining of 2 days of epigastric abdominal pain. Temperature is 100.3°F, BP 100/60 mmHg, and pulse 119/min. He is in distress, and has abdominal pain with tenderness in all four quadrants. WBC count is 21,000 with 91% neutrophils. CT scan of the abdomen shows a severely dilated appendix with a significant amount of fluid concerning for perforation. The next step in the management is
Correct
Incorrect
Explanation:
Open appendectomy via lower midline laparotomy incision. The diagnosis of perforated appendicitis is given in the question, and the goal of the question is straightforward: what is the proper management of perforated appendicitis? The first decision in any scenario is how aggressive to be, i.e. conservative medical management vs. aggressive surgical management. In this scenario, the patient is obviously sick-fever, tachycardia and early hypotension in an immunocompromised patient is self explanatory for not choosing conservative management. Therefore, (A) and (B) are wrong, although they retain a place in the management of non perforated acute appendicitis. If there is known pelvic fluid, the goal of the operation, in addition to removal of the appendix, is to prevent a post-operative, intra-abdominal abscess, i.e. source control. This is done with extensive irrigation of the entire intra-abdominal cavity and breaking up any purulent ovulations that may already exist. This is best done through a midline incision (C) to adequately access both lower quadrants and deep in the pelvis. Alternatively, the laparoscopic approach is excellent for full abdominal visualization and irrigation, but not yet standard of care enough for the purposes of this exam. A standard Mc Burney´s (D) or Pfannesteil incision (E) would severely limit exposure and therefore sufficient irrigation of the abdomen.
-
Question 60 of 100
60. Question
1 points65-year-old diabetic with a sigmoid colectomy three days back has dizziness and faintness. His blood pressure is 82/52 mmHg, pulse is 111 bpm, respiratory rate is 24/min, and SpO2 of 99% on air. His plasma glucose concentration is 18mmol/L (3.0-6.0 fasting).What is the most appropriate investigation?
Correct
Incorrect
Explanation:
This man may have a cardiac cause for his dizziness.The highest prevalence of myocardial infarction (MI) is 72 hours post-operation. Patients with diabetes may not have chest pain due to autonomic dysfunction.
The differential diagnosis would include pulmonary embolus.
It may also include diabetic ketoacidosis, but this would be unlikely with his glucose at 18nmol/L and would not directly explain his hypotension. Also, he would be expected to have a slightly higher respiratory rate than 24/min.
The most appropriate immediate investigation in this scenario would be ECG -
Question 61 of 100
61. Question
1 pointsA patient is involved in a motor vehicle accident (MVA). He presents to the ER, during physical exam you notice he has blood at urinary meatus.
Which of the following is contraindicated at this time?Correct
Incorrect
Explanation:
Transurethral Catheterization is contraindicated in patients who have urethral injuries. Trauma to the urethra should be suspected when a patient has blood at the urethral meatus. A patient who presents with this should be investigated with a retrourethrogram.
-
Question 62 of 100
62. Question
1 pointsA 22 year old man has been involved in a motor vehicle collision. He has blunt chest and abdominal injuries, but the striking feature is his marked restlessness. Which most important initial step should be taken?
Correct
Incorrect
Explanation:
This patient has just had a motor vehicle accident with obvious chest trauma. Any such patient that presents with symptoms of extreme dyspnea, restlessness, anxiety, agitation and decreased breath sounds should be evaluated immediately for a tension pneumothorax with a chest x-ray.
A tension pneumothorax occurs secondary to blunt or penetrating injury of the lung which results in a one-way valve being created. Air leaks from the lung out into the pleural space and is unable to escape, resulting in increased intrapleural pressure. The most immediate goal in the treatment of tension pneumothorax is to alleviate the increased intrapleural pressure. This is most quickly accomplished by performing thoracocentesis using a 20 g needle in the 7th or 8th intercostal space. Once the pleural space has been evacuated, if pneumothorax recurs or is persistent, a tube thoracostomy should be performed. -
Question 63 of 100
63. Question
1 pointsA 35 year old man who weighs 165 pounds and is 5´10″, fractures his wrist. He will need surgical repair and the anesthesiologist decides to do a Bier block (regional IV anesthesia). See picture:
Which of the following is true about this method of anesthesia?Correct
Incorrect
Explanation:
Bier block anesthesia involves lidocaine and a tourniquet. It is indicated for fractures of the forearm, wrist and hand. Bier block is not appropriate for fractures about the elbow, and may not be appropriate in patients with excessive obesity (w/ a large fatty arm).
In this procedure, torniquet is placed on the arm and lidocaine is injected into the dorsal vein in hand, distal to the fracture site. After 30-45 min, most of the lidocaine has been bound to tissues in forearm; therefore, removing the tourniquet at this time does not release a large dose of lidocaine into the general circulation.
Bupivacaine can last for up to 20 hours after surgery and would not be appropriate since after the tourniquet is removed the drug would go into the circulation. A shorter acting drug such as lidoocaine is appropriate as it lasts only 1-5 hours. -
Question 64 of 100
64. Question
1 pointsA 57 year old male, known case of cirrhosis comes to the hospital because he has been vomiting blood. Which diagnostic procedure is most useful in identifying the bleeding source?
Correct
Incorrect
Explanation:
Varices are dilated veins in the distal esophagus or proximal stomach caused by elevated pressure in the portal venous system, typically from cirrhosis. They may bleed massively but cause no other symptoms. Diagnosis is by upper endoscopy (Esophagogastroduodenoscopy or EGD). Treatment is primarily with endoscopic banding and IV octreotide. Sometimes a transjugular intrahepatic portosystemic shunting procedure is needed
-
Question 65 of 100
65. Question
1 pointsA man aged 25 presents with a history of recent bilious vomiting followed by a large volume of bright red hematemesis. He is pale, sweaty and anxious and complains of mild epigastric pain. Blood pressure is 80/50 mm Hg, pulse 128/minute. All of the following steps are correct, except
Correct
Incorrect
Explanation:
Esophaegal varices are dilated veins in the distal esophagus or proximal stomach caused by elevated pressure in the portal venous system, typically from cirrhosis. They may bleed massively but cause no other symptoms. Diagnosis is by upper endoscopy.
Treatment involves fluid resuscitation, endoscopic banding (sclerotherapy second choice) IV octreotide and possibly a transjugular intrahepatic portosystemic shunting (TIPS) procedure.
If bleeding continues or recurs despite these measures, emergency techniques to shunt blood from the portal system to the vena cava can lower portal pressure and diminish bleeding. A TIPS procedure is the emergency intervention of choice. TIPS is an invasive radiologic procedure in which a guidewire is passed from the vena cava through the liver parenchyma into the portal circulation.
The resultant passage is dilated by a balloon catheter, and a metallic stent is inserted, creating a bypass between the portal and hepatic venous circulations. Stent size is crucial. If the stent is too large, hepatic encephalopathy results because of diversion of too much portal blood flow from the liver. If the stent is too small, it is more likely to occlude. -
Question 66 of 100
66. Question
1 pointsFollowing are the surgical indications for diverticulitis except
Correct
Incorrect
Explanation:
Diverticulitis is inflammation of a diverticulum, which can result in phlegmon of the bowel wall, peritonitis, perforation, fistula, or abscess. The primary symptom is abdominal pain. Diagnosis is by CT scan. Treatment is with antibiotics (ciprofloxacin, or a 3rd-generation cephalosporin plus metronidazole) and occasionally surgery.
Surgery is required immediately for patients with free perforation or general peritonitis and for patients with severe symptoms that do not respond to nonsurgical treatment within 48 hours. Increasing pain, tenderness, and fever are other signs that surgery is needed. Surgery > should also be considered in patients with any of the following: 2 2-previous attacks of mild diverticulitis (or one attack in a patient < 50); a persistent tender mass; clinical, endoscopic, or x-ray signs suggestive of cancer; and dysuria associated with diverticulitis in men (or in women who have had a hysterectomy), because this symptom may presage perforation into the bladder. -
Question 67 of 100
67. Question
1 pointsA patient presents with an ulcer on the sole of her foot. X- Ray shows necrosis of part of the 2nd and 3rd metatarsal bone. What is the appropriate management?
Correct
Incorrect
Explanation:
Necrotizing subcutaneous infection is typically caused by a mixture of aerobic and anaerobic organisms that cause necrosis of subcutaneous tissue, usually including the fascia. This infection most commonly affects the extremities and perineum. Affected tissues become red, hot and swollen, resembling severe cellulitis. Without timely treatment, the area becomes gangrenous. Patients are acutely ill. Diagnosis is by history and examination and is supported by evidence of overwhelming infection. Treatment involves antibiotics and surgical debridement. Prognosis is poor without early, aggressive treatment. -
Question 68 of 100
68. Question
1 pointsA man presents with an abscess situated in the right posterior triangle. After its surgical incision the patient could no longer raise her right hand above her head to brush her hair. Which one of the following nerves is injured in this patient?
Correct
Incorrect
Explanation:
The accessory nerve (CN XI) runs through the posterior triangle, so it is likely to be injured in posterior neck triangle operations. If the accessory nerve is damaged there, the trapezius will be denervated, the tip of the shoulder will be drooping and the patient will have inability to raise her hand above her head.
-
Question 69 of 100
69. Question
1 pointsAll of the following are surgically curable causes of hypertension, except
Correct
Incorrect
Explanation:
Coarctation of the aorta is localized narrowing of the aortic lumen that results in upper-extremity hypertension. Treatment is balloon angioplasty with stent placement, or surgical correction.
Renal artery occlusion is acute or chronic interruption of blood flow through one or both of the main renal arteries or its branches, usually due to thromboemboli, atherosclerosis, or fibromuscular dysplasia.
Chronic, progressive occlusion causes refractory hypertension and may lead to chronic renal failure. Treatment of chronic, progressive occlusion includes angioplasty with stenting, surgical bypass, and removal of an infarcted kidney.
A pheochromocytoma is a catecholamine-secreting tumor of chromaffin cells typically located in the adrenals. It causes persistent or paroxysmal hypertension. Treatment involves surgical removal of the tumor when possible.
Essential hypertension, which comprises 90-95% of all cases of hypertension, occurs through no secondary cause. It is treated with lifestyle modifications, diet, exercise and medications. -
Question 70 of 100
70. Question
1 pointsA 46 year old female presents with marked erosion of the nail bed on her toe. A year ago she first noticed an ulcer under her big toe that has progressed to the present state. What step would you take? Correct
Incorrect
Explanation:
Treatment of ischemic ulcerations is mainly supportive through conservative debridement of only necrotic tissue and local wound care to help prevent infection. The prognosis for healing of the ulceration is poor, however, if the arterial perfusion to the area cannot be improved through vascular surgery. Other adjunctive procedures that have been employed with variable success are hyperbaric oxygen therapy and the use of local and systemic vasodilators. These modalities usually are not effective in occlusive disease because the affected area usually is maximally vasodilated secondary to the presence of the local by-products of ischemia.
-
Question 71 of 100
71. Question
1 pointsAcute intestinal obstruction is characterized by all of the following EXCEPT
Correct
Incorrect
Explanation:
Approximately 70- 80% of intestinal gas consists of swallowed air. This air consists primarily of nitrogen, which is poorly absorbed by the intestinal lumen. Impairment of venous return results in severe intestinal congestion, with blood seepage into the intestinal lumen. Accumulation of fluid proximal to an obstructing lesion results from ingested fluid, swallowed saliva, gastric juice, biliary and pancreatic secretions, as well as from interference with normal sodium and water transport. A closed-loop obstruction, such as occurs with an adhesive band, results when the lumen is occluded at two points by the same mechanism.
-
Question 72 of 100
72. Question
1 pointsA patient in severe congestive heart failure develops a flocculent, 10-cm scrotal mass over a week´s period. Transillumination of the scrotum reveals is large fluid collection anterior to the testis, on the left. Which anatomical structure is most likely involved by this mass?
Correct
Incorrect
Explanation:
The patient has developed large hydrocele, in which there is accumulation of clear fluid (that probably communicates with ascites fluid in the abdomen) in the tunica vaginalis. Other causes of hydrocele include infection, tumor and systemic edema related to renal disease. Infants with congenital hydrocele usually have a failure of closure of the process vaginalis, but adults who develop a “new” hydrocele typically have a previously normal or near normal process vaginalis that is “dissected” by the fluid. Blood in the tunica vaginalis (due to trauma or hemorrhagic disease) causes a hematocele. Lymphatic fluid in the tunica vaginalis causes a chylocele, which produces the massively enlarged scrotum observed in patients with elephantisis. Hydrocele can compress the epididymis (choice A) or testes (choice C), but does not usually directly involve these structures. The more distant prostate (choice B) and seminal vesicle (choice E) are usually not affected by hydrocele.
-
Question 73 of 100
73. Question
1 pointsA 31-year-old non-obese female is diagnosed with pheochromocytoma. Which one of the following is contraindicated in this disease?
Correct
Incorrect
Explanation:
This woman has pheochromocytoma, which can be sporadic or familial. Hypertension is paroxysmal in 50% of cases and other symptoms are labile also. The cause of symptoms is excess catecholamines secreted by the tumor, which is usually located in the adrenal glands. Surgical option requires prior treatment with the non-specific and irreversible alpha adrenoceptor blocker Phenoxybenzamine (Irreversible blockade is important because a massive release of catecholamines from the tumor may overcome a reversible blockade). Doing so permits the surgery to proceed while minimizing the likelihood of severe intraoperative hypertension (as might occur when the tumor is manipulated). Some authorities would recommend that a combined alpha/beta blocker such as labetalol also be given in order to slow the heart rate. Regardless, a “pure” beta blocker such as atenolol must never be used in the presence of a pheochromocytoma due to the risk of such treatment´s leading to unopposed alpha agonism and, thus, severe and potentially refractory hypertension. It can only be done after prior alpha antagonization. Clonidine suppression tests are one of the diagnostic tests of choice to rule out pheochromocytoma. 0.3 mg of clonidine given to a patient with pheochromocytoma will cause a rise in catecholamines. Imaging studies, such as CT scan or MRI of the abdomen, can detect as many as 90% of the tumors if the tumors are at least 1 cm in size.
-
Question 74 of 100
74. Question
1 pointsAn 18 year old boy develops a painless, firm mass beneath the nipple of his left breast. The mass is mobile, and no fluid can be expresses from the breast. The right breast is normal to examination. This mass most likely represents which one of the following?
Correct
Incorrect
Explanation:
The most common breast mass in men, especially under 25 years of age, is gynecomastia, which is a benign proliferation of ductal and stromal elements of the breast. It is generally an idiopathic condition, probably related to pubertal hormonal changes. Gynecomastia can involve either one or both breasts and, if it arises during adolescence1wilI often resolve spontaneously with time. Fibrocystic changes in the breast, which reflect physiological responses in the breast to cyclical levels of sex hormones, are not observed in men. Fibrocystic changes are most common in the late reproductive years and include fibrosis cyst formation, and a variety of epithelial changes such as hyperplasia and apocrine metaplasia. Intraductal papillomas are benign neoplasms commonly evolving in the major lactiferous ducts beneath the nipple. They most commonly present with a bloody nipple discharge and are rare in men. Carcinoma of the male breast is rare and almost always develops in the breasts of elderly men. Male breast carcinomas have a somewhat worse prognosis than their female counterpart. Grossly and microscopically they usually resemble ductal carcinoma in the female. The male breast does not develop lobules and so carcinomas resembling lobular carcinoma are extremely rare in the male breast.
-
Question 75 of 100
75. Question
1 pointsA patient suffering from Crohn´s disease undergoes resection of the ileum. What vitamin deficiency is most likely to develop in this patient?
Correct
Incorrect
Explanation:
Cyanocobalamin, or vitamin B12, is absorbed in the terminal ileum and, therefore, is the most likely vitamin deficiency to develop in this patient. The other vitamins are less likely to be deficient.
-
Question 76 of 100
76. Question
1 pointsA person develops an incarcerated loop of bowel within the hernia sac. This is described as a
Correct
Incorrect
Explanation:
A hernia is an abnormal protrusion of tissue through a fascial defect. A Spigelian hernia is an acquired hernia through the linea semilunaris (the lateral border of the rectus abdominus muscle). A Richter hernia occurs when part of the bowel becomes incarcerated in the defect. A direct hernia occurs because of a defect in the transversalis fascia in Hesselbach´s triangle, as opposed to an indirect inguinal hernia, in which the processus vaginalis fails to be obliterated. Littre´s hernia contains a Meckel´s diverticulum within it. A pantaloon hernia is a combined direct and indirect hernia.
-
Question 77 of 100
77. Question
1 pointsA 57 year old woman with breast cancer presents with sudden shortness of breath, orthopnea, pedal edema, and elevation of jugular veins. Rales were auscultated on the lung exam. She had been undergoing a course of chemotherapy for the last six months. This patient´s present problem can be most likely due to
Correct
Incorrect
Explanation:
Doxorubicin is an anthracycline. It is associated with myofibrillar loss and subsequent cardiomyopathy if given for a cumulative dose of 450 mm2 body surface area. Cardiotoxicity can be prevented by dexrazoxane. An echocardiogram and MUGA scan can evaluate the extent of systolic ventricular dysfunction. This patient has congestive heart failure secondary to cardiomyopathy from doxorubicin toxicity. Cisplastin is given for ovarian cancer and it causes renal toxicity as well as bone marrow toxicity. Cyclophosphamide is an alkylating agent used for many leukemias and solid tumors. It causes severe neutropenia but it is known to cause bladder toxicity (hemorrhagic cystitis). Bladder toxicity can be blocked by administration of the uroprotective agent mesna. Amphotericin B is used to treat fungal infections, and it causes renal toxicity. Methotrexate causes liver toxicity.
-
Question 78 of 100
78. Question
1 pointsA 19 year old girl is diagnosed with Hodgkin´s disease. In the future she wishes to have children. Radiotherapy is the recommended treatment. What should be the appropriate management at this time?
Correct
Incorrect
Explanation:
Laparotomy allows accurate staging and allows for transposition of the ovaries out of the radiation field, which is desirable if the patient wishes children in the future. Treatment should not be delayed, as this could lead to further spread of the disease. Chemotherapy will prevent a future healthy pregnancy. Progesterone will have no protective effect.
-
Question 79 of 100
79. Question
1 pointsIntravenous fluid resucitation and care in a burn unit is necessary for burns that cover more than how much percent of the body?
Correct
Incorrect
Explanation:
IV fluid resucitation and care in a burn unit is necessary if the bums cover more than 25% of the body. More aggressive grafting and escharotomies are often necessary in this setting and require units specially equipped to handle these patients. -
Question 80 of 100
80. Question
1 pointsA 6 year old girl has been in the emergency room several times in the last few months for various types of trauma, including a fractured fibula, clavicle, and radius as noted by the emergency room resident. According to the mother the child had injured herself playing. The child is now brought in with a femoral fracture. What should the resident do?
Correct
Incorrect
Explanation:
A child who is brought to an emergency department multiple times with trauma is likely to be the victim of child abuse. Emergency room personnel must be aware of this possibility and evaluate it appropriately. A bone marrow biopsy would not be helpful. Individuals with osteogenesis imperfecta do not usually present in this fashion. Myeloma is very unlikely in a child. Casting a child such as this and sending her home puts her at risk for more abuse, unless the issue of child abuse has been evaluated and ruled out.
-
Question 81 of 100
81. Question
1 pointsA 73 year old nonsmoker and non drinker man has problem with maintaining an erection. He is able to achieve erection, but is not able to sustain it for sufficient time. He takes atenolol, isosorbide dinitrate, lisinopril, furosemide, ibuprofen, and omeprazole. His vitals are normal and BMI is 24. Labs are normal. His testosterone level is 435ng/dL. Appropriate recommendation for him is
Correct
Incorrect
Explanation:
Phosphodiesterase inhibitors such as sildenafil (C) are the first line therapy for erectile dysfunction (ED). However, this patient cannot take sildenafil because of the concurrent use of nitrates. The second line therapy for ED that should be employed in this patient is a vacuum device. Penile implants (A) should be used as a last resort for those who fail all other conventional therapy. Sertraline (B) is an antidepressant. This patient´s history is not suggestive of depression; furthermore, sertraline can cause sexual problems, including erectile dysfunction and retarded ejaculation, which makes it contraindicated in this case. Testosterone (D) is not necessary, as the patient´s testosterone level is normal.
-
Question 82 of 100
82. Question
1 pointsA 61 year old diabetic and hypertensive woman has diarrhea and 30 lb weight loss in last 3 months. Her stools are bulky and foul smelling. She also has hypercholesterolemia, coronary artery disease and end stage renal disease. She is undernourished. Vitals and systemic examination is normal. Abdomen CT scan reveals at 2cm mass at the head of pancreas consistent with pancreatic cancer. Liver shows evidence of metastasis. The appropriate step in management is
Correct
Incorrect
Explanation:
Pancreatic cancer has very high mortality. . When associated with metastasis, the mortality rate is 100%. In the setting of metastasis, aggressive management by doing surgery is not recommended. Instead, treatment should be supportive with:
Pain management
Control of other symptoms like diarrhea
Placement of a biliary stent in case of biliary obstruction resulting in pruritus Symptomatic relief with cholecystectomy
Chemotherapy (A) and radiotherapy (B) are generally used as adjuvant therapies after surgery in patients with resectable pancreatic cancer. They have also been used in combination in locally invasive disease. However, they do not have a role in treatment of metastatic pancreatic cancer. A Whipple procedure (D) (pancreatico-duodenectomy) is performed only if the pancreatic cancer is resectable and there is no metastasis in patients who are considered to be fit for surgery. It should be noted that a Whipple procedure is associated with a high intraoperative mortality and is generally not performed in patients with a lot of comorbidities. A Whipple procedure is generally not performed when there is evidence of metastasis. Therefore, resection of part of the liver (E) in combination with a Whipple procedure is not done. -
Question 83 of 100
83. Question
1 pointsA 24 year old male is shot with a 22 caliber revolver. The entrance wound is in the anteromedial aspect of his upper thigh, and the exit wound is about 3 inches lower, in the posterolateral aspect of the thigh. He has a large, expanding hematoma in the upper inner thigh. No palpable pulses are present in the foot. The bone is intact by exam and X-ray films. Appropriate next step in management is Correct
Incorrect
Explanation:
This one should be obvious. There is no question that the femoral vessels are injured, and the expanding hematoma plus absent pulses indicate that the femoral artery is involved (the vein may or may not be injured). Surgical exploration starts with proximal and distal control. Once the hematoma is safely entered, the extent of the injuries can be ascertained and the proper repair done. Doppler studies (A) are wonderful when we are trying to ascertain whether blood is flowing through a vessel. The clinical findings here are obvious. No fancy technology is needed. Venograms (B) are not needed prior to surgery. We can look at the vein during the operation. Arteriograms (C) are very often used in vascular trauma, but are not needed here. We would use an arteriogram if the anatomic location of the injury suggested vascular involvement, but the clinical signs did not confirm such suspicion. Arteriograms are also used when the specific surgical approach is dictated by precise knowledge of the site of extravasation, a situation that does not apply here. An embolectomy (D) will probably be done at the end of the surgical procedure, as a routine part of removing debris from the lumen of a vessel that has been repaired or re-anastomosed. However, embolectomy alone is the wrong answer for this vignette. The absent pulses are due to disruption of the artery, not to embolization from a distant source
-
Question 84 of 100
84. Question
1 pointsA 68 year old smoker and drinker male has progressive jaundice, which he first noticed 6 weeks ago. His total billirubin is 22 mg/dL. AST and ALT are minimally elevated, but ALP is approximately 6 times the upper limit of normal. Sonogram shows dilated intrahepatic ducts, dilated extrahepatic ducts, and much distended, thin walled gallbladder without stones. Appropriate next step in diagnosis is
Correct
Incorrect
Explanation:
Obstructive jaundice is evident by the high alkaline phosphatase and the dilated biliary ducts. Malignancy is suggested by the dilated, thin walled gallbladder without stones. If there is a cancer of the head of the pancreas, CT scan has a good chance of showing it in a noninvasive manner. Endoscopic retrograde cholangiopancreatography (ERCP) (B) or percutaneous transhepatic cholangiogram (PTC) (D) would be the next step if the CT scan were negative. A smaller tumor at the ampulla, the common duct itself, or even the head of the pancreas could escape detection by the CT scan and would necessitate a more invasive study to show up. Either of these could be used, although ERCP is favored by most practitioners. Exploratory laparotomy (C) would be premature at this point. Serologies (E) would have been called for if he had very high transaminases, normal or near normal alkaline phosphatase, and an unremarkable sonogram.
-
Question 85 of 100
85. Question
1 pointsA 34 year old man noticed a mass in scrotum a week ago. The mass is painless. Physical exam is normal. Palpation of his scrotum reveals a smooth painless mass, just above his testicle that transilluminates when light is shined through it. Scrotal ultrasound reveals cystic lesion without internal echoes in the right epididymis. The next step in management is
Correct
Incorrect
Explanation:
The patient has a simple cyst in the head of his right epididymis. The differential diagnosis is limited, and consists of a spermatocele versus an epididymis cyst. The difference is the presence or absence of sperm within the cyst. As long as the patient is asymptomatic, no further follow-up or management is necessary, as the cyst will eventually involute. Fine-needle aspiration (A) is unnecessary and potentially dangerous, as spilled sperm within the scrotum can incite an inflammatory response. Follow-up with future ultrasounds (B) is unnecessary, as the lesion is benign. Staging CT scan (D) is also unnecessary, as the lesion is benign. Surgical excision (E) is incorrect. If the cyst was causing pain, then surgical excision would be appropriate.
-
Question 86 of 100
86. Question
1 pointsA 67 year old woman presents to the ER with an incarcerated umbilical hernia. She has gross abdominal distention, is dehydrated, and reports persistent fecaloid vomiting for the past 3 days. She is tired, weak, and thirsty, but is awake and alert. Labs reveal a sodium concentration of 118mEq/L. She does not have any chronic medical conditions. The physiologic explanation for the serum sodium is
Correct
Incorrect
Explanation:
Gastrointestinal tract fluids have a sodium concentration very close to that of plasma; as they are lost (internally or externally), they should be replaced with isotonic, sodium containing fluids. But that is not what patients typically do at home. Thirsty and unable to eat solid (sodium-containing) foods, they drink water, Coke, and tea, fluids without significant amounts of sodium, which the body avidly retains because of the severe volume depletion. Endogenous water from catabolic activity is also retained. Dilutional hyponatremia eventually develops. She does not have “water intoxication” (A). This term denotes abnormal water retention due to excessive water infusion at a time when there is a high level of ADH in the blood. This patient is retaining water because she is desperately volume depleted, not because high volumes of water are being forced into her.
The hyponatremia is not due to the loss of hypertonic fluid (B). There are no hypertonic fluids in the gut or anywhere else for that matter. The only hypertonic fluid that we can lose is highly concentrated urine, but we usually do so as a physiologic response to save water. Yes, we often see comatose and convulsing patients when they have this much hyponatremia (D), but that happens when water retention is massive and fast. Slow water retention allows the brain to adapt. One can see even lower serum sodium concentrations in patients with a clear sensorial. Volume deprivation leads to renal wasting of sodium (E) is plain wrong. What the kidney does when there is volume depletion is to increase reabsorption of sodium, not to dump it. -
Question 87 of 100
87. Question
1 pointsA 65 year old woman presents for her annual visit. She has been well. Medical history is significant for osteopenia, for which she takes calcium and vitamin D supplements. On exam, a mass is palpated in right breast. Mammogram shows a mass in the inner upper quadrant of the right breast, and she is scheduled for an ultrasound guided biopsy. Which one favors a better prognosis in her?
Correct
Incorrect
Explanation:
Dual positivity for estrogen and progesterone receptors predicts improved response to endocrine treatment, time to relapse, and overall survival. The presence of the estrogen receptor on the tumor allows for treatment with tamoxifen, a competitive inhibitor of estrogen, which significantly reduces mortality and risk for recurrence. The accepted prognostic markers that provide critical information necessary for treatment decisions are TNM stage, axillary lymph node status, tumor size and grade, and hormone receptor status. Other good prognostic factors include:
1. Stage I cancer (less than 2 cm in size and has not spread from the breast)
2. No lymph node involvement. The status of the axillary lymph nodes is one of the most useful prognostic indicators for breast cancer, with average 10 year survival rates dropping drastically in node-positive patients.
3. Carcinoma in situ
4. Medullary, lobular, and mucinous carcinomas have a slightly better prognosis then invasive ductal carcinoma (B)
5. Positivity for ER and PR hormone receptors
The poor prognostic factors include:
l. Large size tumors (D)
2. Carcinoma with axillary lymph node involvement
3. Higher grade carcinoma
4. ER or PR negativity (C) on immunohistochemical staining
5. Overexpression of HER2/neu
Among all the choices given, (E) has the best prognosis for this patient -
Question 88 of 100
88. Question
1 pointsA 20-year-old woman sustains severe head injuries in a road traffic accident.Investigations are normal except that sodium and chloride are elevated.Which one of the following statements is correct?
Correct
Incorrect
Explanation:
There is a marked hypernatraemia with elevated chloride but normal potassium and urea in a patient with severe head injuries. The likely cause of this presentation is diabetes insipidus. Urine osmolality is therefore likely to be low. You cannot say that she has a hyperchloraemic acidosis as you do not have her bicarbonate concentration but if you assume that the anion gap would be normal, that is 10-12, then this would suggest that the bicarbonate is elevated, suggesting either a metabolic alkalosis or respiratory acidosis with compensation. Although restoration of normal volume and osmolality is required, giving 5% dextrose may exacerbate any cerebral oedema and so correction should be gradual.
-
Question 89 of 100
89. Question
1 pointsFollowing a Hartmanns procedure, a 76-year-old man with ischaemic heart disease is in High Dependency unit. He is in pain and has put 100/200/300 ml of blood into his drains in last 3 hours. Blood pressure is 110/80 mmHg, pulse is 105 beats/min, a respiratory rate of 32/min, SpO2 100% on oxygen by face mask.
Haemoglobin 8.1 g/dL (13.0-18.0) White Cell Count 4.5X109/L (4-11X109)
Platelets 132X109/L (150-400X109) Which of the following is the next most appropriate therapeutic intervention?
Correct
Incorrect
Explanation:
This is a difficult question as all the answers seem reasonable! His observations could be caused by pain. However in the face of a falling haemoglobin, a rising pulse rate and an increasing loss into his drains this patient needs to return to theatre. However his raised pulse rate and a low haemoglobin suggest resuscitation and they are not severe enough to warrant going straight to theatre without intervention. His blood pressure is not critically low and although many would give crystalloid, it is not unreasonable in an elderly man with ischaemic heart disease to get blood if it is easily available.
-
Question 90 of 100
90. Question
1 pointsAn elderly lady with breast cancer is starting diamorphine elixir for painful bony metastases. Which of the following is the most appropriate comment to make to her caregiver?
Correct
Incorrect
Explanation:
Sedation occurring in the first few days typically wears off.
A laxative should be used along and started in conjunction with narcotics to avoid distressing constipation.
If pain relief is inadequate the dose should be increased; cocaine may produce hallucinations.
Addiction is not an issue in the terminally ill.
IM is 3 times more effective than the same oral dose. -
Question 91 of 100
91. Question
1 pointsWhat is the most common cause of intestinal obstruction in males over 65 years
Correct
Incorrect
Explanation:
The most common cause of SBO is postsurgical adhesions. Postoperative adhesions can be the cause of acute obstruction within 4 weeks of surgery or of chronic obstruction decades later. The incidence of SBO parallels the increasing number of laparotomies performed in developing countries.
The second most common identified cause of SBO is an incarcerated groin hernia. Other etiologies of SBO include malignant tumor (20%), hernia (10%), inflammatory bowel disease (5%), volvulus (3%), and miscellaneous causes (2%). The causes of SBO in pediatric patients include congenital atresia, pyloric stenosis, and intussusception. -
Question 92 of 100
92. Question
1 pointsA 64 year old male is on his second postoperative day after colectomy for carcinoma. He is on intravenous fluids and nasogastric suction. His state of hydration can be assessed by which one of the following?
Correct
Incorrect
Explanation:
Hydration status can be evaluated by various measures. For example checking the eyeballs for sunkenness, mucous membranes, heart rate, skin turgor, capillary refill time, and urine output. Close monitoring of urine output is most essential and should be maintained in the early post-operative period.
-
Question 93 of 100
93. Question
1 pointsA male aged 29 is admitted to the emergency department with a faciomaxillary injury and inability to breathe. In the absence of special equipment, an airway is best made through which one of the following structures?
Correct
Incorrect
Explanation:
In a trauma case it is always essential to keep the airway clear. It may be necessary to carry out procedures to do this under various conditions. The cricothyroid membrane. A large calibre needle may be passed though the cricothyroid membrane to provide an essential airway when the upper airway is obstructed. The cricothyroid membrane fills in the space between the anterior cricoid cartilage and the thyroid cartilage. This and other procedures on the larynx and trachea must be performed strictly in the mid-line to avoid large blood vessels. -
Question 94 of 100
94. Question
1 pointsA white mail patient aged 58 years has been in ICU for the past 10 days recovering from an exploratory laparotomy performed for a perforated duodenal ulcer. The patient developed acute renal failure and sepsis postoperatively. 3 days ago the patient became hypothermic and the resident on duty ordered a thyroid function tests and obtained the following results:
T4RIA …………………………….. 4 microg/dL (N 5-12)
T3RIA ………………………………. 60 n/dL /N 70-190)
TSH ………………………………… 2.0 microU/mL (N 0.5-5.0)No past history of thyroid disease is present. His gland is normal in size. His condition today is critical but stable. The most appropriate management at this time is to
Correct
Incorrect
Explanation:
This patient in all probability has the euthyroid-sick syndrome, also known as nonthyroid illness syndrome (NTI)-the association of severe nonthyroidal illness with biochemical parameters indicative of thyroid hypofunction. Low T3 with normal T4 and low T3 with low T4 are the most common variants of this syndrome. TSH is usually normal but may be high or low. A TSH level > 20 microU/mL would be inconsistent with NTI and ´indicates hypothyroidism. Replacement of T4 does not influence the outcome.
-
Question 95 of 100
95. Question
1 pointsA patient is brought to the ER following an accident. On arrival he is in a state of hypovolemic shock. A STAT CT of the abdomen shows partial tears in the spleen. What is the most appropriate management?
Correct
Incorrect
Explanation:
Splenic rupture generally results from blunt abdominal trauma. Significant impact (eg, motor vehicle collision) can rupture a normal spleen. Rupture of the splenic capsule produces marked hemorrhage into the peritoneal cavity. The manifestations, including hemorrhagic shock, abdominal pain, and distention, are usually clinically obvious. However, splenic trauma can also produce a subcapsular hematoma, which may not rupture until hours or even months after the injury.
Rupture is generally preceded by left upper quadrant abdominal pain. Splenic rupture should be suspected in patients with blunt abdominal trauma and hemorrhagic shock or left upper quadrant pain (which sometimes radiates to the shoulder); patients with unexplained left upper quadrant pain, particularly if there is evidence of hypovolemia or shock, should be asked about recent trauma. The diagnosis is confirmed by CT scan (in the stable patient), ultrasound, or peritoneal lavage (in the unstable patient). Treatment has traditionally been splenectomy. However, splenectomy should be avoided if possible, particularly in children, to avoid the resulting permanent susceptibility to bacterial infections. In this case, treatment is transfusion as needed. -
Question 96 of 100
96. Question
1 pointsA 52 year old lady comes to you because she has found a breast mass. On examination you feel a firm, fixed, nontender, 2cm mass. No discharge is present and axillary lymph nodes are not palpable. You send her for a mammogram, and then obtain cells for cytologic examination via fine needle aspiration. The mammogram is read as “suspicious” and the fine-needle cytology report reads, “a few benign ductal epithelioid cells and adipose tissue”. The most appropriate next step in management is which one of the following?
Correct
Incorrect
Explanation:
In the ideal setting, the accuracy of fine-needle aspiration may be over 90%. Clinical information is critical in interpreting the results of fine- needle aspiration – more so than with a tissue biopsy because of the more limited sampling. It is crucial to determine whether the findings on fine-needle aspiration explain the clinical findings. Although the report from the mammogram and the biopsy are not ominous, they do not explain the clinical findings in this case. Immediate repeat fine-needle aspiration or, even better, a tissue biopsy is indicated. Proceeding directly to therapy, whether surgery or irradiation, is inappropriate because the diagnosis is not clearly established. Likewise, any delay in establishing the diagnosis is not appropriate.
-
Question 97 of 100
97. Question
1 pointsA one day post operative abdominal surgery patient has fever of 101.5° F. Medications consist of morphine and sc heparin. One dose of cefazolin was given before the surgery and another dose immediately after surgery. The patient has no known drug allergies and has been on these medications previously. What is the most likely cause for this fever?
Correct
Incorrect
Explanation:
The most likely cause of fever in this patient is post operative atelectasis, which can be effectively treated with good pulmonary toilet, i.e., incentive spirometry and deep breathing. Urinary tract infection usually will not become apparent until post operative day three. This commonly occurs as a result of catheterization preoperatively or colonization of the catheter post-operatively.
-
Question 98 of 100
98. Question
1 pointsA 42 year old man presents with testicular mass and gynecomastia. Testicular biopsy was performed that revealed the presence of crystals of Reinke. The most likely diagnosis is
Correct
Incorrect
Explanation:
While Sertoli and Leydig cell tumors may present with testicular mass and gynecomastia, only Leydig cell tumors exhibit crystals of Reinke within the cytoplasm of tumorigenic cells. None of the other choices typically present with palpable testicular neoplasms as the first symptoms.
-
Question 99 of 100
99. Question
1 pointsWhat is the best management of an unstable patient with a colonic obstruction?
Correct
Incorrect
Explanation:
Cecostomy is a relatively minor procedure and is the treatment of choice in unstable patients with large bowel obstruction. Delaying surgery could result in perforation. Hemicolectomy, staged or unstaged and ilea rectal anastomosis are major surgical procedures and should not be undertaken in unstable patients.
-
Question 100 of 100
100. Question
1 pointsA colon cancer and five surrounding lymph nodes are biopsied. Histopathology shows that the lesion extends into the muscularis propria with two positive lymph nodes. What is the Aster Coller modification of Duke´s staging system?
Correct
Incorrect
Explanation:
The Aster Coller modification of Duke´s staging system is commonly used to stage colon carcinoma. Stage A is when the tumor does not extend into the muscularis. Stage B1 is when the tumor extends into the muscularis without lymph node involvement, and stage C1 is stage B1 with lymph node involvement. Stage B2 is when the tumor extends past the muscularis without lymph node involvement, and stage C2 is stage B2 with lymph node involvement. Stage D is distant metastatic disease.